N330 test 2

Réussis tes devoirs et examens dès maintenant avec Quizwiz!

What finding on a prenatal visit at 10 weeks could suggest a hydatidiform mole? a.Complaint of frequent mild nausea b.Blood pressure of 120/80 mm Hg c.Fundal height measurement of 18 cm d.History of bright red spotting for 1 day, weeks ago

ANS: C The uterus in a hydatidiform molar pregnancy is often larger than would be expected on the basis of the duration of the pregnancy. Nausea increases in a molar pregnancy because of the increased production of hCG. A woman with a molar pregnancy may have early-onset pregnancy-induced hypertension. In the patients history, bleeding is normally described as brownish.

A pregnant womans diet may not meet her need for folates. A good source of this nutrient is: a.Chicken b.Cheese c.Potatoes d.Green leafy vegetables

ANS: D Sources of folates include green leafy vegetables, whole grains, fruits, liver, dried peas, and beans. Chicken and cheese are excellent sources of protein but are poor in folates. Potatoes contain carbohydrates and vitamins and minerals but are poor in folates.

The nurse recognizes that a nonstress test (NST) in which two or more fetal heart rate (FHR) accelerations of 15 beats/min or more occur with fetal movement in a 20-minute period is: a.Nonreactive b.Positive c.Negative d.Reactive

ANS: D The NST is reactive (normal) when two or more FHR accelerations of at least 15 beats/min (each with a duration of at least 15 seconds) occur in a 20-minute period. A nonreactive result means that the heart rate did not accelerate during fetal movement. A positive result is not used with NST. Contraction stress test (CST) uses positive as a result term. A negative result is not used with NST. CST uses negative as a result term.

A woman currently uses a diaphragm and spermicide for contraception. She asks the nurse what the major differences are between the cervical cap and diaphragm. The nurses most appropriate response is: a.No spermicide is used with the cervical cap, so its less messy. b.The diaphragm can be left in place longer after intercourse. c.Repeated intercourse with the diaphragm is more convenient. d.The cervical cap can safely be used for repeated acts of intercourse without adding more spermicide later.

ANS: D The cervical cap can be inserted hours before sexual intercourse without the need for additional spermicide later. No additional spermicide is required for repeated acts of intercourse. Spermicide should be used inside the cap as an additional chemical barrier. The cervical cap should remain in place for 6 hours after the last act of intercourse. Repeated intercourse with the cervical cap is more convenient because no additional spermicide is needed.

A woman in the active phase of the first stage of labor is using a shallow pattern of breathing, which is about twice the normal adult breathing rate. She starts to complain about feeling lightheaded and dizzy and states that her fingers are tingling. The nurse should: a.Notify the womans physician. b.Tell the woman to slow the pace of her breathing. c.Administer oxygen via a mask or nasal cannula. d.Help her breathe into a paper bag

ANS: D This woman is experiencing the side effects of hyperventilation, which include the symptoms of lightheadedness, dizziness, tingling of the fingers, or circumoral numbness. Having the woman breathe into a paper bag held tightly around her mouth and nose may eliminate respiratory alkalosis. This enables her to rebreathe carbon dioxide and replace the bicarbonate ion.

To determine the cultural influence on a patients diet, the nurse should first: a.Evaluate the patients weight gain during pregnancy b.Assess the socioeconomic status of the patient c.Discuss the four food groups with the patient d.Identify the food preferences and methods of food preparation common to that culture

ANS: D Understanding the patients food preferences and how she prepares food will assist the nurse in determining whether the patients culture is adversely affecting her nutritional intake. Evaluation of a patients weight gain during pregnancy should be included for all patients, not just for patients who are culturally different. The socioeconomic status of the patient may alter the nutritional intake but not the cultural influence. Teaching the food groups to the patient should come after assessing food preferences.

In planning for the care of a 30-year-old woman with pregestational diabetes, the nurse recognizes that the most important factor affecting pregnancy outcome is the: a.Mothers age. b.Number of years since diabetes was diagnosed. c.Amount of insulin required prenatally. d.Degree of glycemic control during pregnancy.

ANS: D Women with excellent glucose control and no blood vessel disease should have good pregnancy outcomes.

Which vitamins or minerals can lead to congenital malformations of the fetus if taken in excess by the mother? a.Zinc b.Vitamin D c.Folic acid d.Vitamin A

ANS: D Zinc, vitamin D, and folic acid are vital to good maternal and fetal health and are highly unlikely to be consumed in excess. Vitamin A taken in excess causes a number of problems. An analog of vitamin A appears in prescribed acne medications, which must not be taken during pregnancy.

A placenta previa in which the placental edge just reaches the internal os is more commonly known as: a.Total b.Partial c.Complete d.Marginal

ANS: D A placenta previa that does not cover any part of the cervix is termed marginal. With a total placenta previa, the placenta completely covers the os. When the patient experiences a partial placenta previa, the lower border of the placenta is within 3 cm of the internal cervical os but does not completely cover the os. A complete placenta previa is termed total. The placenta completely covers the internal cervical os.

Evaluation for infertility should be offered to couples who have failed to become pregnant after 1 year of regular intercourse or after 6 months if the woman is older than 35. Impaired fertility in women may be the result of numerous factors. Careful identification of the cause of infertility assists in determining the correct treatment plan. The nurse who chooses to work in the specialty of infertility must have an excellent understanding of these factors and causes. Match each factor affecting female infertility with the likely cause. a.Ovarian b.Tubal/peritoneal c.Uterine d.Vaginal/cervical e.Other factors 35. Endometrial or myometrial tumors 36. Anorexia 37. Isoimmunization 38. Thyroid dysfunction or obesity 39. Endometriosis

ANS: 35. C 36. A 37. D 38. E 39. B

22. Which occurrence is associated with cervical dilation and effacement? a.Bloody show b.False labor c.Lightening d.Bladder distention

ANS: A As the cervix begins to soften, dilate, and efface, expulsion of the mucous plug that sealed the cervix during pregnancy occurs. This causes rupture of small cervical capillaries. Cervical dilation and effacement do not occur with false labor. Lightening is the descent of the fetus toward the pelvic inlet before labor. Bladder distention occurs when the bladder is not emptied frequently. It may slow down the descent of the fetus during labor.

16. Compared with contraction stress test (CST), nonstress test (NST) for antepartum fetal assessment: a.Has no known contraindications. b.Has fewer false-positive results. c.Is more sensitive in detecting fetal compromise. d.Is slightly more expensive.

ANS: A CST has several contraindications. NST has a high rate of false-positive results, is less sensitive than the CST, and is relatively inexpensive.

Which presentation is described accurately in terms of both presenting part and frequency of occurrence? a.Cephalic: occiput; at least 95% b.Breech: sacrum; 10% to 15% c.Shoulder: scapula; 10% to 15% d.Cephalic: cranial; 80% to 85%

ANS: A In cephalic presentations (head first), the presenting part is the occiput; this occurs in 96% of births. In a breech birth, the sacrum emerges first; this occurs in about 3% of births. In shoulder presentations, the scapula emerges first; this occurs in only 1% of births

To care for a laboring woman adequately, the nurse understands that the __________ stage of labor varies the most in length? a.First b.Second c.Third d.Fourth

ANS: A The first stage of labor is considered to last from the onset of regular uterine contractions to full dilation of the cervix. The first stage is much longer than the second and third stages combined. In a first-time pregnancy the first stage of labor can take up to 20 hours. The second stage of labor lasts from the time the cervix is fully dilated to the birth of the fetus. The average length is 20 minutes for a multiparous woman and 50 minutes for a nulliparous woman. The third stage of labor lasts from the birth of the fetus until the placenta is delivered. This stage may be as short as 3 minutes or as long as 1 hour. The fourth stage of labor, recovery, lasts about 2 hours after delivery of the placenta.

Transvaginal ultrasonography is often performed during the first trimester. While preparing your 6-week gestation patient for this procedure, she expresses concerns over the necessity for this test. The nurse should explain that this diagnostic test may be indicated for a number of situations (Select all that apply). a.Multifetal gestation b.Obesity c.Fetal abnormalities d.Amniotic fluid volume e.Ectopic pregnancy

ANS: A, B, C, E Transvaginal ultrasound is useful in obese women whose thick abdominal layers cannot be penetrated with traditional abdominal ultrasound. This procedure is also used for identifying multifetal gestation, ectopic pregnancy, estimating gestational age, confirming fetal viability, and identifying fetal abnormalities. Amniotic fluid volume is assessed during the second and third trimester. Conventional ultrasound would be used.

Which factors influence cervical dilation (Select all that apply) ? a.Strong uterine contractions b.The force of the presenting fetal part against the cervix c.The size of the female pelvis d.The pressure applied by the amniotic sac e.Scarring of the cervix

ANS: A, B, D, E Dilation of the cervix occurs by the drawing upward of the musculofibrous components of the cervix, which is caused by strong uterine contractions. Pressure exerted by the amniotic fluid while the membranes are intact or by the force applied by the presenting part also can promote cervical dilation. Scarring of the cervix as a result of a previous infection or surgery may slow cervical dilation. Pelvic size does not affect cervical dilation.

Signs that precede labor include (Select all that apply): a.Lightening. b.Exhaustion. c.Bloody show. d.Rupture of membranes. e.Decreased fetal movement.

ANS: A, C, D Signs that precede labor may include lightening, urinary frequency, backache, weight loss, surge of energy, bloody show, and rupture of membranes. Many women experience a burst of energy before labor. A decrease in fetal movement is an ominous sign that does not always correlate with labor.

To assist the woman after delivery of the infant, the nurse knows that the blood patch is used after spinal anesthesia to relieve: a. Hypotension. b. Headache. c. Neonatal respiratory depression. d. Loss of movement.

ANS: B The subarachnoid block may cause a postspinal headache resulting from loss of cerebrospinal fluid from the puncture in the dura. When blood is injected into the epidural space in the area of the dural puncture, it forms a seal over the hole to stop leaking of cerebrospinal fluid. Hypotension is prevented by increasing fluid volume before the procedure. Neonatal respiratory depression is not an expected outcome with spinal anesthesia. Loss of movement is an expected outcome of spinal anesthesia.

In terms of the incidence and classification of diabetes, maternity nurses should know that: a.Type 1 diabetes is most common. b.Type 2 diabetes often goes undiagnosed. c.Gestational diabetes mellitus (GDM) means that the woman will be receiving insulin treatment until 6 weeks after birth. d.Type 1 diabetes may become type 2 during pregnancy.

ANS: B Type 2 diabetes often goes undiagnosed because hyperglycemia develops gradually and often is not severe. Type 2 diabetes, sometimes called adult onset diabetes, is the most common. GDM refers to any degree of glucose intolerance first recognized during pregnancy. Insulin may or may not be needed. People do not go back and forth between types 1 and 2 diabetes.

Which meal would provide the most absorbable iron? a.Toasted cheese sandwich, celery sticks, tomato slices, and a grape drink b.Oatmeal, whole wheat toast, jelly, and low-fat milk c.Black bean soup, wheat crackers, orange sections, and prunes d.Red beans and rice, cornbread, mixed greens, and decaffeinated tea

ANS: C Food sources that are rich in iron include liver, meats, whole grain or enriched breads and cereals, deep green leafy vegetables, legumes, and dried fruits. In addition, the vitamin C in orange sections aids absorption. Dairy products and tea are not sources of iron.

The labor of a pregnant woman with preeclampsia is going to be induced. Before initiating the Pitocin infusion, the nurse reviews the womans latest laboratory test findings, which reveal a platelet count of 90,000, an elevated aspartate transaminase (AST) level, and a falling hematocrit. The nurse notifies the physician because the laboratory results are indicative of: a.Eclampsia. b.Disseminated intravascular coagulation (DIC). c.HELLP syndrome. d.Idiopathic thrombocytopenia.

ANS: C HELLP syndrome is a laboratory diagnosis for a variant of severe preeclampsia that involves hepatic dysfunction characterized by hemolysis (H), elevated liver enzymes (EL), and low platelets (LP). Eclampsia is determined by the presence of seizures. DIC is a potential complication associated with HELLP syndrome. Idiopathic thrombocytopenia is the presence of low platelets of unknown cause and is not associated with preeclampsia.

33. To manage her diabetes appropriately and ensure a good fetal outcome, the pregnant woman with diabetes will need to alter her diet by: a.Eating six small equal meals per day. b.Reducing carbohydrates in her diet. c.Eating her meals and snacks on a fixed schedule. d.Increasing her consumption of protein.

ANS: C Having a fixed meal schedule will provide the woman and the fetus with a steadier blood sugar level, provide better balance with insulin administration, and help prevent complications. It is more important to have a fixed meal schedule than equal division of food intake. Approximately 45% of the food eaten should be in the form of carbohydrates.

The nurse providing care for the antepartum woman should understand that contraction stress test (CST): a.Sometimes uses vibroacoustic stimulation. b.Is an invasive test; however, contractions are stimulated. c.Is considered negative if no late decelerations are observed with the contractions. d.Is more effective than nonstress test (NST) if the membranes have already been ruptured.

ANS: C No late decelerations is good news. Vibroacoustic stimulation is sometimes used with NST. CST is invasive if stimulation is by intravenous oxytocin but not if by nipple stimulation and is contraindicated if the membranes have ruptured.

When nurses help their expectant mothers assess the daily fetal movement counts, they should be aware that: a.Alcohol or cigarette smoke can irritate the fetus into greater activity. b.Kick counts should be taken every half hour and averaged every 6 hours, with every other 6-hour stretch off. c.The fetal alarm signal should go off when fetal movements stop entirely for 12 hours. d.Obese mothers familiar with their bodies can assess fetal movement as well as average-size women.

ANS: C No movement in a 12-hour period is cause for investigation and possibly intervention. Alcohol and cigarette smoke temporarily reduce fetal movement. The mother should count fetal activity (kick counts) two or three times daily for 60 minutes each time. Obese women have a harder time assessing fetal movement.

Identify a goal of a patient with the following nursing diagnosis: Imbalanced Nutrition: Less Than Body Requirements related to diet choices inadequate to meet nutrient requirements of pregnancy. a.Gain a total of 30 lb. b.Take daily supplements consistently. c.Decrease intake of snack foods. d.Increase intake of complex carbohydrates.

ANS: A A weight gain of 30 lb is one indication that the patient has gained a sufficient amount for the nutritional needs of pregnancy. A daily supplement is not the best goal for this patient. It does not meet the basic need of proper nutrition during pregnancy. Decreasing snack foods may be a problem and should be assessed; however, assessing weight gain is the best method of monitoring nutritional intake for this pregnant patient. Increasing the intake of complex carbohydrates is important for this patient, but monitoring the weight gain should be the end goal.

The priority nursing intervention when admitting a pregnant woman who has experienced a bleeding episode in late pregnancy is to: a.Assess fetal heart rate (FHR) and maternal vital signs b.Perform a venipuncture for hemoglobin and hematocrit levels c.Place clean disposable pads to collect any drainage d.Monitor uterine contractions

ANS: A Assessment of the FHR and maternal vital signs will assist the nurse in determining the degree of the blood loss and its effect on the mother and fetus. The most important assessment is to check mother/fetal well-being. The blood levels can be obtained later. It is important to assess future bleeding; however, the top priority remains mother/fetal well-being. Monitoring uterine contractions is important but not the top priority.

Approximately 10% to 15% of all clinically recognized pregnancies end in miscarriage. Which is the most common cause of spontaneous abortion? a.Chromosomal abnormalities b.Infections c.Endocrine imbalance d.Immunologic factors

ANS: A At least 50% of pregnancy losses result from chromosomal abnormalities that are incompatible with life. Maternal infection may be a cause of early miscarriage. Endocrine imbalances such as hypothyroidism or diabetes are possible causes for early pregnancy loss. Women who have repeated early pregnancy losses appear to have immunologic factors that play a role in spontaneous abortion incidents.

A pregnant woman reports that she is still playing tennis at 32 weeks of gestation. The nurse would be most concerned that during and after tennis matches this woman consumes: a.Several glasses of fluid. b.Extra protein sources such as peanut butter. c.Salty foods to replace lost sodium. d.Easily digested sources of carbohydrate.

ANS: A If no medical or obstetric problems contraindicate physical activity, pregnant women should get 30 minutes of moderate physical exercise daily. Liberal amounts of fluid should be consumed before, during, and after exercise because dehydration can trigger premature labor. The womans calorie intake should be sufficient to meet the increased needs of pregnancy and the demands of exercise.

In planning care for women with preeclampsia, nurses should be aware that: a.Induction of labor is likely, as near term as possible. b.If at home, the woman should be confined to her bed, even with mild preeclampsia. c.A special diet low in protein and salt should be initiated. d.Vaginal birth is still an option, even in severe cases

ANS: A Induction of labor is likely, as near term as possible; however, at less than 37 weeks of gestation, immediate delivery may not be in the best interest of the fetus. Strict bed rest is becoming controversial for mild cases; some women in the hospital are even allowed to move around. Diet and fluid recommendations are much the same as for healthy pregnant women, although some authorities have suggested a diet high in protein. Women with severe preeclampsia should expect a cesarean delivery.

A woman is 16 weeks pregnant and has elected to terminate her pregnancy. The nurse knows that the most common technique used for medical termination of a pregnancy in the second trimester is: a.Dilation and evacuation (D&E). b.Instillation of hypertonic saline into the uterine cavity. c.Intravenous administration of Pitocin. d.Vacuum aspiration.

ANS: A The most common technique for medical termination of a pregnancy in the second trimester is D&E. It is usually performed between 13 and 16 weeks. Hypertonic solutions injected directly into the uterus account for less than 1% of all abortions because other methods are safer and easier to use. Intravenous administration of Pitocin is used to induce labor in a woman with a third-trimester fetal demise. Vacuum aspiration is used for abortions in the first trimester.

Which major neonatal complication is carefully monitored after the birth of the infant of a diabetic mother? a.Hypoglycemia b.Hypercalcemia c.Hypobilirubinemia d.Hypoinsulinemia

ANS: A The neonate is at highest risk for hypoglycemia because fetal insulin production is accelerated during pregnancy to metabolize excessive glucose from the mother. At birth, the maternal glucose supply stops and the neonatal insulin exceeds the available glucose, thus leading to hypoglycemia. Hypocalcemia is associated with preterm birth, birth trauma, and asphyxia, all common problems of the infant of a diabetic mother. Excess erythrocytes are broken down after birth and release large amounts of bilirubin into the neonates circulation, with resulting hyperbilirubinemia. Because fetal insulin production is accelerated during pregnancy, the neonate presents with hyperinsulinemia.

A pregnant womans biophysical profile score is 8. She asks the nurse to explain the results. The nurses best response is: a.The test results are within normal limits. b.Immediate delivery by cesarean birth is being considered. c.Further testing will be performed to determine the meaning of this score. d.An obstetric specialist will evaluate the results of this profile and, within the next week, will inform you of your options regarding delivery.

ANS: A The normal biophysical score ranges from 8 to 10 points if the amniotic fluid volume is adequate. A normal score allows conservative treatment of high-risk patients. Delivery can be delayed if fetal well-being is indicated. Scores less than 4 should be investigated, and delivery could be initiated sooner than planned. This score is within normal range, and no further testing is required at this time. The results of the biophysical profile are usually available immediately after the procedure is performed.

The patient that you are caring for has severe preeclampsia and is receiving a magnesium sulfate infusion. You become concerned after assessment when the woman exhibits: a.A sleepy, sedated affect. b.A respiratory rate of 10 breaths/min. c.Deep tendon reflexes of 2. d.Absent ankle clonus.

ANS: B A respiratory rate of 10 breaths/min indicates that the client is experiencing respiratory depression from magnesium toxicity. Because magnesium sulfate is a central nervous system depressant, the client will most likely become sedated when the infusion is initiated. Deep tendon reflexes of 2 and absent ankle clonus are normal findings.

Which statement made by a lactating woman would lead the nurse to believe that the woman might have lactose intolerance? a.I always have heartburn after I drink milk. b.If I drink more than a cup of milk, I usually have abdominal cramps and bloating. c.Drinking milk usually makes me break out in hives. d.Sometimes I notice that I have bad breath after I drink a cup of milk.

ANS: B Abdominal cramps and bloating are consistent with lactose intolerance. One problem that can interfere with milk consumption is lactose intolerance, which is the inability to digest milk sugar because of a lack of the enzyme lactose in the small intestine. Milk consumption may cause abdominal cramping, bloating, and diarrhea people who are lactose intolerant, although many affected individuals can tolerate small amounts of milk without symptoms.

What laboratory marker is indicative of disseminated intravascular coagulation (DIC)? a.Bleeding time of 10 minutes b.Presence of fibrin split products c.Thrombocytopenia d.Hyperfibrinogenemia

ANS: B Degradation of fibrin leads to the accumulation of fibrin split products in the blood. Bleeding time in DIC is normal. Low platelets may occur with but are not indicative of DIC because they may result from other coagulopathies. Hypofibrinogenemia would occur with DIC.

A patient with pregnancy-induced hypertension is admitted complaining of pounding headache, visual changes, and epigastric pain. Nursing care is based on the knowledge that these signs are an indication of: a.Anxiety due to hospitalization. b.Worsening disease and impending convulsion. c.Effects of magnesium sulfate. d.Gastrointestinal upset.

ANS: B Headache and visual disturbances are caused by increased cerebral edema. Epigastric pain indicates distention of the hepatic capsules and often warns that a convulsion is imminent. These are danger signs showing increased cerebral edema and impending convulsion and should be treated immediately. The patient has not been started on magnesium sulfate treatment yet. Also, these are not anticipated effects of the medication.

With regard to spinal and epidural (block) anesthesia, nurses should know that: a.This type of anesthesia is commonly used for cesarean births but is not suitable for vaginal births. b.A high incidence of after-birth headache is seen with spinal blocks. c.Epidural blocks allow the woman to move freely. d.Spinal and epidural blocks are never used together.

ANS: B Headaches may be prevented or mitigated to some degree by a number of methods. Spinal blocks may be used for vaginal births, but the woman must be assisted through labor. Epidural blocks limit the womans ability to move freely. Combined use of spinal and epidural blocks is becoming increasingly popular

Maternal nutritional status is an especially significant factor of the many factors that influence the outcome of pregnancy because: a.It is very difficult to adjust because of peoples ingrained eating habits. b.It is an important preventive measure for a variety of problems. c.Women love obsessing about their weight and diets. d.A womans preconception weight becomes irrelevant.

ANS: B Nutritional status draws so much attention not only for its effect on a healthy pregnancy and birth but also because significant changes are within relatively easy reach.

A new mother with which of these thyroid disorders would be strongly discouraged from breastfeeding? a.Hyperthyroidism b.Phenylketonuria (PKU) c.Hypothyroidism d.Thyroid storm

ANS: B PKU is a cause of mental retardation in infants; mothers with PKU pass on phenylalanine. A woman with hyperthyroidism or hypothyroidism would have no particular reason not to breastfeed. A thyroid storm is a complication of hyperthyroidism.

The most prevalent clinical manifestation of abruptio placentae (as opposed to placenta previa) is: a.Bleeding. b.Intense abdominal pain. c.Uterine activity. d.Cramping.

ANS: B Pain is absent with placenta previa and may be agonizing with abruptio placentae. Bleeding may be present in varying degrees for both placental conditions. Uterine activity and cramping may be present with both placental conditions.

Risk factors tend to be interrelated and cumulative in their effect. While planning the care for a laboring client with diabetes mellitus, the nurse is aware that she is at a greater risk for: a.Oligohydramnios. b.Polyhydramnios. c.Postterm pregnancy. d.Chromosomal abnormalities.

ANS: B Polyhydramnios (amniotic fluid >2000 mL) is 10 times more likely to occur in diabetic compared with nondiabetic pregnancies. Polyhydramnios puts the mother at risk for premature rupture of membranes, premature labor, and postpartum hemorrhage. Prolonged rupture of membranes, intrauterine growth restriction, intrauterine fetal death, and renal agenesis (Potter syndrome) all put the client at risk for developing oligohydramnios. Anencephaly, placental insufficiency, and perinatal hypoxia all contribute to the risk for postterm pregnancy. Maternal age older than 35 and balanced translocation (maternal and paternal) are risk factors for chromosome abnormalities.

A woman presents to the emergency department with complaints of bleeding and cramping. The initial nursing history is significant for a last menstrual period 6 weeks ago. On sterile speculum examination, the primary care provider finds that the cervix is closed. The anticipated plan of care for this woman would be based on a probable diagnosis of which type of spontaneous abortion? a.Incomplete b.Inevitable c.Threatened d.Septic

ANS: C A woman with a threatened abortion presents with spotting, mild cramps, and no cervical dilation. A woman with an incomplete abortion would present with heavy bleeding, mild to severe cramping, and cervical dilation. An inevitable abortion manifests with the same symptoms as an incomplete abortion: heavy bleeding, mild to severe cramping, and cervical dilation. A woman with a septic abortion presents with malodorous bleeding and typically a dilated cervix.

Pregnant adolescents are at high risk for _____ because of lower body mass indices (BMIs) and fad dieting. a.Obesity b.Diabetes c.Low-birth-weight babies d.High-birth-weight babies

ANS: C Adolescents tend to have lower BMIs because they are still developing and may follow unsafe nutritional practices. In addition, the fetus and still-growing mother appear to compete for nutrients. These factors, along with inadequate weight gain, lend themselves to a higher incidence of low-birth-weight babies. Obesity, diabetes, and high-birth-weight babies are conditions associated with higher BMIs.

Injectable progestins (DMPA, Depo-Provera) are a good contraceptive choice for women who: a.Want menstrual regularity and predictability. b.Have a history of thrombotic problems or breast cancer. c.Have difficulty remembering to take oral contraceptives daily. d.Are homeless or mobile and rarely receive health care.

ANS: C Advantages of DMPA include a contraceptive effectiveness comparable to that of combined oral contraceptives with the requirement of only four injections a year. Disadvantages of injectable progestins are prolonged amenorrhea and uterine bleeding. Use of injectable progestin carries an increased risk of venous thrombosis and thromboembolism. To be effective, DMPA injections must be administered every 11 to 13 weeks. Access to health care is necessary to prevent pregnancy or potential complications.

At 35 weeks of pregnancy a woman experiences preterm labor. Tocolytics are administered and she is placed on bed rest, but she continues to experience regular uterine contractions, and her cervix is beginning to dilate and efface. What would be an important test for fetal well-being at this time? a.Percutaneous umbilical blood sampling (PUBS) b.Ultrasound for fetal size c.Amniocentesis for fetal lung maturity d.Nonstress test (NST)

ANS: C Amniocentesis would be performed to assess fetal lung maturity in the event of a preterm birth. Indications for PUBS include prenatal diagnosis or inherited blood disorders, karyotyping of malformed fetuses, detection of fetal infection, determination of the acid-base status of a fetus with intrauterine growth restriction, and assessment and treatment of isoimmunization and thrombocytopenia in the fetus. Typically, fetal size is determined by ultrasound during the second trimester and is not indicated in this scenario. NST measures the fetal response to fetal movement in a noncontracting mother.

A laboring woman received an opioid agonist (meperidine) intravenously 90 minutes before she gave birth. Which medication should be available to reduce the postnatal effects of Demerol on the neonate? a.Fentanyl (Sublimaze) b.Promethazine (Phenergan) c.Naloxone (Narcan) d.Nalbuphine (Nubain)

ANS: C An opioid antagonist can be given to the newborn as one part of the treatment for neonatal narcosis, which is a state of central nervous system (CNS) depression in the newborn produced by an opioid. Opioid antagonists such as naloxone (Narcan) can promptly reverse the CNS depressant effects, especially respiratory depression. Fentanyl, promethazine, and nalbuphine do not act as opioid antagonists to reduce the postnatal effects of Demerol on the neonate. Although meperidine (Demerol) is a low-cost medication and readily available, the use of Demerol in labor has been controversial because of its effects on the neonate.

When assessing a woman in labor, the nurse is aware that the relationship of the fetal body parts to one another is called fetal: a.Lie. b.Presentation. c.Attitude. d.Position.

ANS: C Attitude is the relation of the fetal body parts to one another. Lie is the relation of the long axis (spine) of the fetus to the long axis (spine) of the mother. Presentation refers to the part of the fetus that enters the pelvic inlet first and leads through the birth canal during labor at term. Position is the relation of the presenting part to the four quadrants of the mothers pelvis

While working with the pregnant woman in her first trimester, the nurse is aware that chorionic villus sampling (CVS) can be performed during pregnancy at: a.4 weeks b.8 weeks c.10 weeks d.14 weeks

ANS: C CVS can be performed in the first or second trimester, ideally between 10 and 13 weeks of gestation. During this procedure, a small piece of tissue is removed from the fetal portion of the placenta. If performed after 9 completed weeks of gestation, the risk of limb reduction is no greater than in the general population.

What condition indicates concealed hemorrhage when the patient experiences an abruptio placentae? a.Decrease in abdominal pain b.Bradycardia c.Hard, boardlike abdomen d.Decrease in fundal height

ANS: C Concealed hemorrhage occurs when the edges of the placenta do not separate. The formation of a hematoma behind the placenta and subsequent infiltration of the blood into the uterine muscle results in a very firm, boardlike abdomen. Abdominal pain may increase. The patient will have shock symptoms that include tachycardia. As bleeding occurs, the fundal height will increase.

While instructing a couple regarding birth control, the nurse should be aware that the method called natural family planning: a.Is the same as coitus interruptus, or pulling out. b.Uses the calendar method to align the womans cycle with the natural phases of the moon. c.Is the only contraceptive practice acceptable to the Roman Catholic church. d.Relies on barrier methods during fertility phases.

ANS: C Natural family planning is another name for periodic abstinence, which is the accepted way to pass safely through the fertility phases without relying on chemical or physical barriers. Natural family planning is the only contraceptive practice acceptable to the Roman Catholic church. Pulling out is not the same as periodic abstinence, another name for natural family planning. The phases of the moon are not part of the calendar method or any method.

The labor and delivery nurse is preparing a bariatric patient for an elective cesarean birth. Which piece of specialized equipment is unnecessary when providing care for this pregnant woman. a.Extra long surgical instruments b.Wide surgical table c.Temporal thermometer d.Increased diameter blood pressure cuff

ANS: C Obstetricians today are seeing more morbidly obese pregnant women weighing 400, 500, and 600 pounds. To manage their conditions and to meet their logistical needs, a new medical subspecialty bariatric obstetrics has arisen. Extra-wide blood pressure cuffs, scales that can accommodate up to 880 pounds, and extra-wide surgical tables designed to hold the weight of these women are used. Special techniques for ultrasound examination and longer surgical instruments for cesarean birth are also required. A temporal thermometer can be used for a pregnant patient of any size.

Which heart condition is not a contraindication for pregnancy? a.Peripartum cardiomyopathy b.Eisenmenger syndrome c.Heart transplant d.All of these contraindicate pregnancy.

ANS: C Pregnancy is contraindicated for peripartum cardiomyopathy and Eisenmenger syndrome. Women who have had heart transplants are successfully having babies. However, conception should be postponed for at least 1 year after transplantation.

The maternity nurse understands that as the uterus contracts during labor, maternal-fetal exchange of oxygen and waste products: a.Continues except when placental functions are reduced. b.Increases as blood pressure decreases. c.Diminishes as the spiral arteries are compressed. d.Is not significantly affected.

ANS: C Uterine contractions during labor tend to decrease circulation through the spiral electrodes and subsequent perfusion through the intervillous space. The maternal blood supply to the placenta gradually stops with contractions. The exchange of oxygen and waste products decreases. The exchange of oxygen and waste products is affected by contractions

With regard to the noncontraceptive medical effects of combined oral contraceptive pills (COCs), nurses should be aware that: a.COCs can cause toxic shock syndrome if the prescription is wrong. b.Hormonal withdrawal bleeding usually is a bit more profuse than in normal menstruation and lasts a week. c.COCs increase the risk of endometrial and ovarian cancer. d.The effectiveness of COCs can be altered by some over-the-counter medications and herbal supplements.

ANS: D The effectiveness of COCs can be altered by some over-the-counter medications and herbal supplements. Toxic shock syndrome can occur in some diaphragm users, but it is not a consequence of taking oral contraceptive pills. Hormonal withdrawal bleeding usually is lighter than in normal menstruation and lasts a couple of days. Oral contraceptive pills offer protection against the risk of endometrial and ovarian cancers.

The obstetric nurse is preparing the patient for an emergency cesarean birth, with no time to administer spinal anesthesia. The nurse is aware and prepared for the greatest risk of administering general anesthesia to the patient. This risk is: a. Respiratory depression. b. Uterine relaxation. c. Inadequate muscle relaxation d. Aspiration of stomach contents.

ANS: D Aspiration of acidic gastric contents with possible airway obstruction is a potentially fatal complication of general anesthesia. Respirations can be altered during general anesthesia, and the anesthesiologist will take precautions to maintain proper oxygenation. Uterine relaxation can occur with some anesthesia; however, this can be monitored and prevented. Inadequate muscle relaxation can be improved with medication.

Your patient has been receiving magnesium sulfate for 20 hours for treatment of preeclampsia. She just delivered a viable infant girl 30 minutes ago. What uterine findings would you expect to observe/assess in this client? a.Absence of uterine bleeding in the postpartum period b.A fundus firm below the level of the umbilicus c.Scant lochia flow d.A boggy uterus with heavy lochia flow

ANS: D Because of the tocolytic effects of magnesium sulfate, this patient most likely would have a boggy uterus with increased amounts of bleeding and a heavy lochia flow in the postpartum period.

As related to the care of the patient with miscarriage, nurses should be aware that: a.It is a natural pregnancy loss before labor begins. b.It occurs in fewer than 5% of all clinically recognized pregnancies. c.It often can be attributed to careless maternal behavior such as poor nutrition or excessive exercise. d.If it occurs before the twelfth week of pregnancy, it may manifest only as moderate discomfort and blood loss.

ANS: D Before the sixth week the only evidence may be a heavy menstrual flow. After the twelfth week more severe pain, similar to that of labor, is likely. Miscarriage is a natural pregnancy loss, but by definition it occurs before 20 weeks of gestation, before the fetus is viable. Miscarriages occur in approximately 10% to 15% of all clinically recognized pregnancies. Miscarriage can be caused by a number of disorders or illnesses outside of the mothers control or knowledge.

The major source of nutrients in the diet of a pregnant woman should be composed of: a.Simple sugars b.Fats c.Fiber d.Complex carbohydrates

ANS: D Complex carbohydrates supply the pregnant woman with vitamins, minerals, and fiber. The most common simple carbohydrate is table sugar, which is a source of energy but does not provide any nutrients. Fats provide 9 kcal in each gram, in contrast to carbohydrates and proteins, which provide only 4 kcal in each gram. Fiber is supplied primarily by complex carbohydrates.

When counseling a client about getting enough iron in her diet, the maternity nurse should tell her that: a.Milk, coffee, and tea aid iron absorption if consumed at the same time as iron. b.Iron absorption is inhibited by a diet rich in vitamin C. c.Iron supplements are permissible for children in small doses. d.Constipation is common with iron supplements.

ANS: D Constipation can be a problem. Milk, coffee, and tea inhibit iron absorption when consumed at the same time as iron. Vitamin C promotes iron absorption. Children who ingest iron can get very sick and even die.

Which statement is true about the term contraceptive failure rate? a.It refers to the percentage of users expected to have an accidental pregnancy over a 5-year span. b.It refers to the minimum level that must be achieved to receive a government license. c.It increases over time as couples become more careless. d.It varies from couple to couple, depending on the method and the users.

ANS: D Contraceptive effectiveness varies from couple to couple, depending on how well a contraceptive method is used and how well it suits the couple. The contraceptive failure rate measures the likelihood of accidental pregnancy in the first year only. Failure rates decline over time because users gain experience.

Since the gene for cystic fibrosis was identified in 1989, data can be collected for the purposes of genetic counseling for couples regarding carrier status. According to statistics, how often does cystic fibrosis occur in Caucasian live births? a.1 in 100 c.1 in 2500 b.1 in 1200 d.1 in 3000

ANS: D Cystic fibrosis occurs in about 1 in 3000 Caucasian live births.

Which of the following statements is the most complete and accurate description of medical abortions? a.They are performed only for maternal health. b.They can be achieved through surgical procedures or with drugs. c.They are mostly performed in the second trimester. d.They can be either elective or therapeutic.

ANS: D Medical abortions are performed through the use of medications (rather than surgical procedures). They are mostly done in the first trimester, and they can be either elective (the womans choice) or therapeutic (for reasons of maternal or fetal health).

What form of heart disease in women of childbearing years usually has a benign effect on pregnancy? a.Cardiomyopathy b.Rheumatic heart disease c.Congenital heart disease d.Mitral valve prolapse

ANS: D Mitral valve prolapse is a benign condition that is usually asymptomatic. Cardiomyopathy produces congestive heart failure during pregnancy. Rheumatic heart disease can lead to heart failure during pregnancy. Some congenital heart diseases produce pulmonary hypertension or endocarditis during pregnancy.

Which method of pain management is safest for a gravida 3 para 2 admitted at 8 cm cervical dilation? a.Epidural anesthesia b.Narcotics c.Spinal block d.Breathing and relaxation techniques

ANS: D Nonpharmacologic methods of pain management may be the best option for a woman in advanced labor. It is unlikely that enough time remains to administer epidural or spinal anesthesia. A narcotic given at this time may reach its peak about the time of birth and result in respiratory depression in the newborn.

In their role of implementing a plan of care for infertile couples, nurses should: a.Be comfortable with their sexuality and nonjudgmental about others to counsel their clients effectively. b.Know about such nonmedical remedies as diet, exercise, and stress management. c.Be able to direct clients to sources of information about what herbs to take that might help and which ones to avoid. d.Do all of the above plus be knowledgeable about potential drug and surgical remedies.

ANS: D Nurses should be open to and ready to help with a variety of medical and nonmedical approaches.

32. Glucose metabolism is profoundly affected during pregnancy because: a.Pancreatic function in the islets of Langerhans is affected by pregnancy. b.The pregnant woman uses glucose at a more rapid rate than the nonpregnant woman. c.The pregnant woman increases her dietary intake significantly. d.Placental hormones are antagonistic to insulin, thus resulting in insulin resistance.

ANS: D Placental hormones, estrogen, progesterone, and human placental lactogen (HPL) create insulin resistance. Insulin also is broken down more quickly by the enzyme placental insulinase. Pancreatic functioning is not affected by pregnancy. The glucose requirements differ because of the growing fetus. The pregnant woman should increase her intake by 200 calories a day.

A pregnant womans diet history indicates that she likes the following list of foods. The nurse would encourage this woman to consume more of which food to increase her calcium intake? a.Fresh apricots b.Canned clams c.Spaghetti with meat sauce d.Canned sardines

ANS: D Sardines are rich in calcium. Fresh apricots, canned clams, and spaghetti with meat sauce are not high in calcium.

A 32-year-old primigravida is admitted with a diagnosis of ectopic pregnancy. Nursing care is based on the knowledge that: a.Bed rest and analgesics are the recommended treatment. b.She will be unable to conceive in the future. c.A D&C will be performed to remove the products of conception. d.Hemorrhage is the major concern.

ANS: D Severe bleeding occurs if the fallopian tube ruptures. The recommended treatment is to remove the pregnancy before rupture in order to prevent hemorrhaging. If the tube must be removed, the womans fertility will decrease; however, she will not be infertile. D&C is performed on the inside of the uterine cavity. The ectopic pregnancy is located within the tubes.

When caring for a pregnant woman with cardiac problems, the nurse must be alert for signs and symptoms of cardiac decompensation, which include: a.A regular heart rate and hypertension. b.An increased urinary output, tachycardia, and dry cough. c.Shortness of breath, bradycardia, and hypertension. d.Dyspnea; crackles; and an irregular, weak pulse.

ANS: D Signs of cardiac decompensation include dyspnea; crackles; an irregular, weak, rapid pulse; rapid respirations; a moist, frequent cough; generalized edema; increasing fatigue; and cyanosis of the lips and nail beds. A regular heart rate and hypertension are not generally associated with cardiac decompensation. Tachycardia would indicate cardiac decompensation, but increased urinary output and a dry cough would not. Shortness of breath would indicate cardiac decompensation, but bradycardia and hypertension would not.

During a physical assessment of an at-risk client, the nurse notes generalized edema, crackles at the base of the lungs, and some pulse irregularity. These are most likely signs of: a.Euglycemia. b.Rheumatic fever. c.Pneumonia. d.Cardiac decompensation.

ANS: D Symptoms of cardiac decompensation may appear abruptly or gradually. Euglycemia is a condition of normal glucose levels. These symptoms indicate cardiac decompensation. Rheumatic fever can cause heart problems, but it does not manifest with these symptoms, which indicate cardiac decompensation. Pneumonia is an inflammation of the lungs and would not likely generate these symptoms, which indicate cardiac decompensation.

12. A woman is using the basal body temperature (BBT) method of contraception. She calls the clinic and tells the nurse, My period is due in a few days, and my temperature has not gone up. The nurses most appropriate response is: a.This probably means that youre pregnant. b.Dont worry; its probably nothing. c.Have you been sick this month? d.You probably didnt ovulate during this cycle.

ANS: D The absence of a temperature decrease most likely is the result of lack of ovulation. Pregnancy cannot occur without ovulation (which is being measured using the BBT method). A comment such as Dont worry; its probably nothing discredits the clients concerns. Illness would most likely cause an increase in BBT.

Prophylaxis of subacute bacterial endocarditis is given before and after birth when a pregnant woman has: a.Valvular disease. c.Arrhythmias. b.Congestive heart disease. d.Postmyocardial infarction.

ANS: A Prophylaxis for intrapartum endocarditis and pulmonary infection may be provided for women who have mitral valve stenosis. Prophylaxis for intrapartum endocarditis is not indicated for congestive heart disease, arrhythmias, or after myocardial infarction.

The nerve block used in labor that provides anesthesia to the lower vagina and perineum is called: a.An epidural. b.A pudendal. c.A local. d.A spinal block

ANS: B A pudendal block anesthetizes the lower vagina and perineum to provide anesthesia for an episiotomy and use of low forceps if needed. An epidural provides anesthesia for the uterus, perineum, and legs. A local provides anesthesia for the perineum at the site of the episiotomy. A spinal block provides anesthesia for the uterus, perineum, and down the legs.

A 39-year-old primigravida thinks that she is about 8 weeks pregnant, although she has had irregular menstrual periods all her life. She has a history of smoking approximately one pack of cigarettes a day, but she tells you that she is trying to cut down. Her laboratory data are within normal limits. What diagnostic technique could be used with this pregnant woman at this time? a.Ultrasound examination b.Maternal serum alpha-fetoprotein (MSAFP) screening c.Amniocentesis d.Nonstress test (NST)

ANS: A An ultrasound examination could be done to confirm the pregnancy and determine the gestational age of the fetus. It is too early in the pregnancy to perform MSAFP screening, amniocentesis, or NST. MSAFP screening is performed at 16 to 18 weeks of gestation, followed by amniocentesis if MSAFP levels are abnormal or if fetal/maternal anomalies are detected. NST is performed to assess fetal well-being in the third trimester.

In vitro fertilization-embryo transfer (IVF-ET) is a common approach for women with blocked fallopian tubes or unexplained infertility and for men with very low sperm counts. A husband and wife have arrived for their preprocedural interview. The husband asks the nurse to explain what the procedure entails. The nurses most appropriate response is: a.IVF-ET is a type of assisted reproductive therapy that involves collecting eggs from your wifes ovaries, fertilizing them in the laboratory with your sperm, and transferring the embryo to her uterus. b.A donor embryo will be transferred into your wifes uterus. c.Donor sperm will be used to inseminate your wife. d.Dont worry about the technical stuff; thats what we are here for.

ANS: A A womans eggs are collected from her ovaries, fertilized in the laboratory with sperm, and transferred to her uterus after normal embryonic development has occurred. The statement, A donor embryo will be transferred into your wifes uterus describes therapeutic donor insemination. Donor sperm will be used to inseminate your wife describes the procedure for a donor embryo. Dont worry about the technical stuff; thats what we are here for discredits the clients need for teaching and is an inappropriate response.

Nursing care measures are commonly offered to women in labor. Which nursing measure reflects application of the gate-control theory? a.Massaging the womans back b.Changing the womans position c.Giving the prescribed medication d.Encouraging the woman to rest between contractions

ANS: A According to the gate-control theory, pain sensations travel along sensory nerve pathways to the brain, but only a limited number of sensations, or messages, can travel through these nerve pathways at one time. Distraction techniques such as massage or stroking, music, focal points, and imagery reduce or completely block the capacity of nerve pathways to transmit pain. These distractions are thought to work by closing down a hypothetic gate in the spinal cord and thus preventing pain signals from reaching the brain. The perception of pain is thereby diminished. Changing the womans position, giving prescribed medication, and encouraging rest do not reduce or block the capacity of nerve pathways to transmit pain using the gate-control theory.

A woman is undergoing a nipple-stimulated contraction stress test (CST). She is having contractions that occur every 3 minutes. The fetal heart rate (FHR) has a baseline of approximately 120 beats/min without any decelerations. The interpretation of this test is said to be: a.Negative. b.Positive. c.Satisfactory. d.Unsatisfactory.

ANS: A Adequate uterine activity necessary for a CST consists of the presence of three contractions in a 10-minute time frame. If no decelerations are observed in the FHR pattern with the contractions, the findings are considered to be negative. A positive CST indicates the presence of repetitive later FHR decelerations. Satisfactory and unsatisfactory are not applicable terms.

A pregnant patient would like to know a good food source of calcium other than dairy products. Your best answer is: a.Legumes b.Yellow vegetables c.Lean meat d.Whole grains

ANS: A Although dairy products contain the greatest amount of calcium, it also is found in legumes, nuts, dried fruits, and some dark green leafy vegetables. Yellow vegetables are rich in vitamin A. Lean meats are rich in protein and phosphorus. Whole grains are rich in zinc and magnesium.

Nurses should be aware that the biophysical profile (BPP): a.Is an accurate indicator of impending fetal death. b.Is a compilation of health risk factors of the mother during the later stages of pregnancy. c.Consists of a Doppler blood flow analysis and an amniotic fluid index. d.Involves an invasive form of ultrasound examination.

ANS: A An abnormal BPP score is an indication that labor should be induced. The BPP evaluates the health of the fetus, requires many different measures, and is a noninvasive procedure.

Spontaneous termination of a pregnancy is considered to be an abortion if: a.The pregnancy is less than 20 weeks. b.The fetus weighs less than 1000 g. c.The products of conception are passed intact. d.No evidence exists of intrauterine infection.

ANS: A An abortion is the termination of pregnancy before the age of viability (20 weeks). The weight of the fetus is not considered because some older fetuses may have a low birth weight. A spontaneous abortion may be complete or incomplete. A spontaneous abortion may be caused by many problems, one being intrauterine infection.

A client asks her nurse, My doctor told me that he is concerned with the grade of my placenta because I am overdue. What does that mean? The best response by the nurse is: a.Your placenta changes as your pregnancy progresses, and it is given a score that indicates the amount of calcium deposits it has. The more calcium deposits, the higher the grade, or number, that is assigned to the placenta. It also means that less blood and oxygen can be delivered to your baby. b.Your placenta isnt working properly, and your baby is in danger. c.This means that we will need to perform an amniocentesis to detect if you have any placental damage. d.Dont worry about it. Everything is fine.

ANS: A An accurate and appropriate response is, Your placenta changes as your pregnancy progresses, and it is given a score that indicates the amount of calcium deposits it has. The more calcium deposits, the higher the grade, or number, that is assigned to the placenta. It also means that less blood and oxygen can be delivered to your baby. Although Your placenta isnt working properly, and your baby is in danger may be valid, it does not reflect therapeutic communication techniques and is likely to alarm the client. An ultrasound, not an amniocentesis, is the method of assessment used to determine placental maturation. The response Dont worry about it. Everything is fine is not appropriate and discredits the clients concerns.

The nurse providing newborn stabilization must be aware that the primary side effect of maternal narcotic analgesia in the newborn is: a.Respiratory depression. b.Bradycardia. c.Acrocyanosis. d.Tachypnea.

ANS: A An infant delivered within 1 to 4 hours of maternal analgesic administration is at risk for respiratory depression from the sedative effects of the narcotic. Bradycardia is not the anticipated side effect of maternal analgesics. Acrocyanosis is an expected finding in a newborn and is not related to maternal analgesics. The infant who is having a side effect to maternal analgesics normally would have a decrease in respirations, not an increase.

Your patient is being induced because of her worsening preeclampsia. She is also receiving magnesium sulfate. It appears that her labor has not become active despite several hours of oxytocin administration. She asks the nurse, Why is it taking so long? The most appropriate response by the nurse would be: a.The magnesium is relaxing your uterus and competing with the oxytocin. It may increase the duration of your labor. b.I dont know why it is taking so long. c.The length of labor varies for different women. d.Your baby is just being stubborn.

ANS: A Because magnesium sulfate is a tocolytic agent, its use may increase the duration of labor. The amount of oxytocin needed to stimulate labor may be more than that needed for the woman who is not receiving magnesium sulfate. I dont know why it is taking so long is not an appropriate statement for the nurse to make. Although the length of labor does vary in different women, the most likely reason this womans labor is protracted is the tocolytic effect of magnesium sulfate. The behavior of the fetus has no bearing on the length of labor.

Marfan syndrome is an autosomal dominant genetic disorder that displays as weakness of the connective tissue, joint deformities, ocular dislocation, and weakness to the aortic wall and root. While providing care to a client with Marfan syndrome during labor, which intervention should the nurse complete first? a.Antibiotic prophylaxis b.b-Blockers c.Surgery d.Regional anesthesia

ANS: A Because of the potential for cardiac involvement during the third trimester and after birth, treatment with prophylactic antibiotics is highly recommended. b-Blockers and restricted activity are recommended as treatment modalities earlier in the pregnancy. Regional anesthesia is well tolerated by clients with Marfan syndrome; however, it is not essential to care. Adequate labor support may be all that is necessary if an epidural is not part of the womans birth plan. Surgery for cardiovascular changes such as mitral valve prolapse, aortic regurgitation, root dilation, or dissection may be necessary. Mortality rates may be as high as 50% in women who have severe cardiac disease.

Because pregnant women may need surgery during pregnancy, nurses should be aware that: a.The diagnosis of appendicitis may be difficult because the normal signs and symptoms mimic some normal changes in pregnancy. b.Rupture of the appendix is less likely in pregnant women because of the close monitoring. c.Surgery for intestinal obstructions should be delayed as long as possible because it usually affects the pregnancy d.When pregnancy takes over, a woman is less likely to have ovarian problems that require invasive responses.

ANS: A Both appendicitis and pregnancy are linked with nausea, vomiting, and increased white blood cell count. Rupture of the appendix is two to three times more likely in pregnant women. Surgery to remove obstructions should be done right away. It usually does not affect the pregnancy. Pregnancy predisposes a woman to ovarian problems.

As related to the care of the patient with anemia, the nurse should be aware that: a.It is the most common medical disorder of pregnancy. b.It can trigger reflex brachycardia. c.The most common form of anemia is caused by folate deficiency. d.Thalassemia is a European version of sickle cell anemia.

ANS: A Combined with any other complication, anemia can result in congestive heart failure. Reflex bradycardia is a slowing of the heart in response to the blood flow increases immediately after birth. The most common form of anemia is iron deficiency anemia. Both thalassemia and sickle cell hemoglobinopathy are hereditary but not directly related or confined to geographic areas.

With regard to nerve block analgesia and anesthesia, nurses should be aware that: a.Most local agents are related chemically to cocaine and end in the suffix -caine. b.Local perineal infiltration anesthesia is effective when epinephrine is added, but it can be injected only once. c.A pudendal nerve block is designed to relieve the pain from uterine contractions. d.A pudendal nerve block, if done correctly, does not significantly lessen the bearing-down reflex

ANS: A Common agents include lidocaine and chloroprocaine. Injections can be repeated to prolong the anesthesia. A pudendal nerve block relieves pain in the vagina, vulva, and perineum but not the pain from uterine contractions, and it lessens or shuts down the bearing-down reflex.

A woman is experiencing back labor and complains of intense pain in her lower back. An effective relief measure would be to use: a.Counterpressure against the sacrum. b.Pant-blow (breaths and puffs) breathing techniques. c.Effleurage. d.Conscious relaxation or guided imagery.

ANS: A Counterpressure is steady pressure applied by a support person to the sacral area with the fist or heel of the hand. This technique helps the woman cope with the sensations of internal pressure and pain in the lower back. The pain management techniques of pant-blow, effleurage, and conscious relaxation or guided imagery are usually helpful for contractions per the gate-control theory.

When the pregnant diabetic woman experiences hypoglycemia while hospitalized, the nurse should intervene by having the patient: a.Eat six saltine crackers. b.Drink 8 oz of orange juice with 2 tsp of sugar added. c.Drink 4 oz of orange juice followed by 8 oz of milk. d.Eat hard candy or commercial glucose wafers.

ANS: A Crackers provide carbohydrates in the form of polysaccharides. Orange juice and sugar will increase the blood sugar but not provide a slow-burning carbohydrate to sustain the blood sugar. Milk is a disaccharide and orange juice is a monosaccharide. They will provide an increase in blood sugar but will not sustain the level. Hard candy or commercial glucose wafers provide only monosaccharides.

Which statement regarding acronyms in nutrition is accurate? a.Dietary reference intakes (DRIs) consist of recommended dietary allowances (RDAs), adequate intakes (AIs), and upper limits (ULs). b.RDAs are the same as ULs except with better data. c.AIs offer guidelines for avoiding excessive amounts of nutrients. d.They all refer to green leafy vegetables, whole grains, and fruit.

ANS: A DRIs consist of RDAs, AIs, and ULs. AIs are similar to RDAs except that they deal with nutrients about which data are insufficient for certainty (RDA status). ULs are guidelines for avoiding excesses of nutrients for which excess is toxic. Green leafy vegetables, whole grains, and fruit are important, but they are not the whole nutritional story.

Which order should the nurse expect for a patient admitted with a threatened abortion? a.Bed rest b.Ritodrine IV c.NPO d.Narcotic analgesia every 3 hours, prn

ANS: A Decreasing the womans activity level may alleviate the bleeding and allow the pregnancy to continue. Ritodrine is not the first drug of choice for tocolytic medications. There is no reason for having the woman placed NPO. At times dehydration may produce contractions, so hydration is important. Narcotic analgesia will not decrease the contractions. It may mask the severity of the contractions.

The nurse sees a woman for the first time when she is 30 weeks pregnant. The woman has smoked throughout the pregnancy, and fundal height measurements now are suggestive of growth restriction in the fetus. In addition to ultrasound to measure fetal size, what other tool would be useful in confirming the diagnosis? a.Doppler blood flow analysis b.Contraction stress test (CST) c.Amniocentesis d.Daily fetal movement counts

ANS: A Doppler blood flow analysis allows the examiner to study the blood flow noninvasively in the fetus and the placenta. It is a helpful tool in the management of high risk pregnancies because of intrauterine growth restriction (IUGR), diabetes mellitus, multiple fetuses, or preterm labor. Because of the potential risk of inducing labor and causing fetal distress, CST is not performed on a woman whose fetus is preterm. Indications for amniocentesis include diagnosis of genetic disorders or congenital anomalies, assessment of pulmonary maturity, and diagnosis of fetal hemolytic disease, not IUGR. Fetal kick count monitoring is performed to monitor the fetus in pregnancies complicated by conditions that may affect fetal oxygenation. Although this may be a useful tool at some point later in this womans pregnancy, it is not used to diagnose IUGR

The nurse would expect which maternal cardiovascular finding during labor? a.Increased cardiac output b.Decreased pulse rate c.Decreased white blood cell (WBC) count d.Decreased blood pressure

ANS: A During each contraction, 400 mL of blood is emptied from the uterus into the maternal vascular system. This increases cardiac output by about 51% above baseline pregnancy values at term. The heart rate increases slightly during labor. The WBC count can increase during labor. During the first stage of labor, uterine contractions cause systolic readings to increase by about 10 mm Hg. During the second stage, contractions may cause systolic pressures to increase by 30 mm Hg and diastolic readings to increase by 25 mm Hg.

Which nutritional recommendation about fluids is accurate? a.A womans daily intake should be eight to ten glasses (2.3 L) of water, milk, or juice. b.Coffee should be limited to no more than two cups, but tea and cocoa can be consumed without worry. c.Of the artificial sweeteners, only aspartame has not been associated with any maternity health concerns. d.Water with fluoride is especially encouraged because it reduces the childs risk of tooth decay.

ANS: A Eight to ten glasses is the standard for fluids; however, they should be the right fluids. All beverages containing caffeine, including tea, cocoa, and some soft drinks, should be avoided or drunk only in limited amounts. Artificial sweeteners, including aspartame, have no ill effects on the normal mother or fetus; however, mothers with phenylketonuria should avoid aspartame. No evidence indicates that prenatal fluoride consumption reduces childhood tooth decay.

Postcoital contraception with Ovral: a.Requires that the first dose be taken within 72 hours of unprotected intercourse. b.Requires that the woman take second and third doses at 24 and 36 hours after the first dose. c.Must be taken in conjunction with an IUD insertion. d.Is commonly associated with the side effect of menorrhagia.

ANS: A Emergency contraception is most effective when used within 72 hours of intercourse; however, it may be used with lessened effectiveness 120 hours later. Insertion of the copper IUD within 5 days of intercourse may also be used and is up to 99% effective. The most common side effect of postcoital contraception is nausea.

With regard to breathing techniques during labor, maternity nurses should understand that: a.Breathing techniques in the first stage of labor are designed to increase the size of the abdominal cavity to reduce friction. b.By the time labor has begun, it is too late for instruction in breathing and relaxation. c.Controlled breathing techniques are most difficult near the end of the second stage of labor. d.The patterned-paced breathing technique can help prevent hyperventilation.

ANS: A First-stage techniques promote relaxation of abdominal muscles, thereby increasing the size of the abdominal cavity. Instruction in simple breathing and relaxation techniques early in labor is possible and effective. Controlled breathing techniques are most difficult in the transition phase at the end of the first stage of labor when the cervix is dilated 8 to 10 cm. Patterned-paced breathing sometimes can lead to hyperventilation.

Which description of the four stages of labor is correct for both definition and duration? a.First stage: onset of regular uterine contractions to full dilation; less than 1 hour to 20 hours b.Second stage: full effacement to 4 to 5 cm; visible presenting part; 1 to 2 hours c.Third state: active pushing to birth; 20 minutes (multiparous women), 50 minutes (first-timer) d.Fourth stage: delivery of the placenta to recovery; 30 minutes to 1 hour

ANS: A Full dilation may occur in less than 1 hour, but in first-time pregnancies it can take up to 20 hours. The second stage extends from full dilation to birth and takes an average of 20 to 50 minutes, although 2 hours is still considered normal. The third stage extends from birth to expulsion of the placenta and usually takes a few minutes. The fourth stage begins after expulsion of the placenta and lasts until homeostasis is reestablished (about 2 hours).

With regard to protein in the diet of pregnant women, nurses should be aware that: a.Many protein-rich foods are also good sources of calcium, iron, and B vitamins. b.Many women need to increase their protein intake during pregnancy. c.As with carbohydrates and fat, no specific recommendations exist for the amount of protein in the diet. d.High-protein supplements can be used without risk by women on macrobiotic diets.

ANS: A Good protein sources such as meat, milk, eggs, and cheese have a lot of calcium and iron. Most women already eat a high-protein diet and do not need to increase their intake. Protein is sufficiently important that specific servings of meat and dairy are recommended. High-protein supplements are not recommended because they have been associated with an increased incidence of preterm births.

A woman with severe preeclampsia has been receiving magnesium sulfate by intravenous infusion for 8 hours. The nurse assesses the woman and documents the following findings: temperature of 37.1 C, pulse rate of 96 beats/min, respiratory rate of 24 breaths/min, blood pressure (BP) of 155/112 mm Hg, 3+ deep tendon reflexes, and no ankle clonus. The nurse calls the physician, anticipating an order for: a.Hydralazine. b.Magnesium sulfate bolus. c.Diazepam. d.Calcium gluconate.

ANS: A Hydralazine is an antihypertensive commonly used to treat hypertension in severe preeclampsia. Typically it is administered for a systolic BP greater than 160 mm Hg or a diastolic BP greater than 110 mm Hg. An additional bolus of magnesium sulfate may be ordered for increasing signs of central nervous system irritability related to severe preeclampsia (e.g., clonus) or if eclampsia develops. Diazepam sometimes is used to stop or shorten eclamptic seizures. Calcium gluconate is used as the antidote for magnesium sulfate toxicity. The client is not currently displaying any signs or symptoms of magnesium toxicity.

In assessing the knowledge of a pregestational woman with type 1 diabetes concerning changing insulin needs during pregnancy, the nurse recognizes that further teaching is warranted when the client states: a.I will need to increase my insulin dosage during the first 3 months of pregnancy. b.Insulin dosage will likely need to be increased during the second and third trimesters. c.Episodes of hypoglycemia are more likely to occur during the first 3 months. d.Insulin needs should return to normal within 7 to 10 days after birth if I am bottle-feeding.

ANS: A Insulin needs are reduced in the first trimester because of increased insulin production by the pancreas and increased peripheral sensitivity to insulin. Insulin dosage will likely need to be increased during the second and third trimesters, Episodes of hypoglycemia are more likely to occur during the first 3 months, and Insulin needs should return to normal within 7 to 10 days after birth if I am bottle-feeding are accurate statements and signify that the woman has understood the teachings regarding control of her diabetes during pregnancy.

Which analysis of maternal serum may predict chromosomal abnormalities in the fetus? a.Multiple-marker screening b.Lecithin/sphingomyelin (L/S) ratio c.Biophysical profile d.Type and crossmatch of maternal and fetal serum

ANS: A Maternal serum can be analyzed for abnormal levels of alpha-fetoprotein, human chorionic gonadotropin, and estriol. The multiple-marker screening may predict chromosomal defects in the fetus. The L/S ratio is used to determine fetal lung maturity. A biophysical profile is used for evaluating fetal status during the antepartum period. Five variables are used, but none is concerned with chromosomal problems. The blood type and crossmatch would not predict chromosomal defects in the fetus.

A woman in active labor receives an analgesic opioid agonist. Which medication relieves severe, persistent, or recurrent pain; creates a sense of well-being; overcomes inhibitory factors; and may even relax the cervix but should be used cautiously in women with cardiac disease? a.Meperidine (Demerol) b.Promethazine (Phenergan) c.Butorphanol tartrate (Stadol) d.Nalbuphine (Nubain)

ANS: A Meperidine is the most commonly used opioid agonist analgesic for women in labor throughout the world. It overcomes inhibitory factors in labor and may even relax the cervix. Because tachycardia is a possible adverse reaction, meperidine is used cautiously in women with cardiac disease. Phenergan is an ataractic (tranquilizer) that may be used to augment the desirable effects of the opioid analgesics but has few of the undesirable effects of those drugs. Stadol and Nubain are opioid agonist-antagonist analgesics.

28. With one exception, the safest pregnancy is one in which the woman is drug and alcohol free. For women addicted to opioids, ________________________ treatment is the current standard of care during pregnancy. a.Methadone maintenance b.Detoxification c.Smoking cessation d.4 Ps Plus

ANS: A Methadone maintenance treatment (MMT) is currently considered the standard of care for pregnant women who are dependent on heroin or other narcotics. Buprenorphine is another medication approved for opioid addiction treatment that is increasingly being used during pregnancy. Opioid replacement therapy has been shown to decrease opioid and other drug use, reduce criminal activity, improve individual functioning, and decrease rates of infections such as hepatitis B and C, HIV, and other sexually transmitted infections. Detoxification is the treatment used for alcohol addiction. Pregnant women requiring withdrawal from alcohol should be admitted for inpatient management. Women are more likely to stop smoking during pregnancy than at any other time in their lives. A smoking cessation program can assist in achieving this goal. The 4 Ps Plus is a screening tool designed specifically to identify pregnant women who need in-depth assessment related to substance abuse.

Which nutrients recommended dietary allowance (RDA) is higher during lactation than during pregnancy? a.Energy (kcal) b.Iron c.Vitamin A d.Folic acid

ANS: A Needs for energy, protein, calcium, iodine, zinc, the B vitamins, and vitamin C remain greater than nonpregnant needs.

Screening at 24 weeks of gestation reveals that a pregnant woman has gestational diabetes mellitus (GDM). In planning her care, the nurse and the woman mutually agree that an expected outcome is to prevent injury to the fetus as a result of GDM. The nurse identifies that the fetus is at greatest risk for: a.Macrosomia. b.Congenital anomalies of the central nervous system. c.Preterm birth. d.Low birth weight.

ANS: A Poor glycemic control later in pregnancy increases the rate of fetal macrosomia. Poor glycemic control during the preconception time frame and into the early weeks of the pregnancy is associated with congenital anomalies. Preterm labor or birth is more likely to occur with severe diabetes and is the greatest risk in women with pregestational diabetes. Increased weight, or macrosomia, is the greatest risk factor for this woman.

The nurse caring for pregnant women must be aware that the most common medical complication of pregnancy is: a.Hypertension. c.Hemorrhagic complications. b.Hyperemesis gravidarum. d.Infections.

ANS: A Preeclampsia and eclampsia are two noted deadly forms of hypertension. A large percentage of pregnant women will have nausea and vomiting, but a relatively few have the severe form called hyperemesis gravidarum. Hemorrhagic complications are the second most common medical complication of pregnancy; hypertension is the most common.

In caring for the woman with disseminated intravascular coagulation (DIC), what order should the nurse anticipate? a.Administration of blood b.Preparation of the client for invasive hemodynamic monitoring c.Restriction of intravascular fluids d.Administration of steroids

ANS: A Primary medical management in all cases of DIC involves correction of the underlying cause, volume replacement, blood component therapy, optimization of oxygenation and perfusion status, and continued reassessment of laboratory parameters. Central monitoring would not be ordered initially in a client with DIC because this can contribute to more areas of bleeding. Management of DIC would include volume replacement, not volume restriction. Steroids are not indicated for the management of DIC.

With regard to the association of maternal diabetes and other risk situations affecting mother and fetus, nurses should be aware that: a.Diabetic ketoacidosis (DKA) can lead to fetal death at any time during pregnancy. b.Hydramnios occurs approximately twice as often in diabetic pregnancies. c.Infections occur about as often and are considered about as serious in diabetic and nondiabetic pregnancies. d.Even mild to moderate hypoglycemic episodes can have significant effects on fetal well-being.

ANS: A Prompt treatment of DKA is necessary to save the fetus and the mother. Hydramnios occurs 10 times more often in diabetic pregnancies. Infections are more common and more serious in pregnant women with diabetes. Mild to moderate hypoglycemic episodes do not appear to have significant effects on fetal well-being.

After you complete your nutritional counseling for a pregnant woman, you ask her to repeat your instructions so you can assess her understanding of the instructions given. Which statement indicates that she understands the role of protein in her pregnancy? a.Protein will help my baby grow. b.Eating protein will prevent me from becoming anemic. c.Eating protein will make my baby have strong teeth after he is born. d.Eating protein will prevent me from being diabetic.

ANS: A Protein is the nutritional element basic to growth. An adequate protein intake is essential to meeting the increasing demands of pregnancy. These demands arise from the rapid growth of the fetus; the enlargement of the uterus, mammary glands, and placenta; the increase in the maternal blood volume; and the formation of amniotic fluid. Iron intake prevents anemia. Calcium intake is needed for fetal bone and tooth development. Glycemic control is needed in diabetics; protein is one nutritional factor to consider, but this is not the primary role of protein intake.

What nursing diagnosis would be the most appropriate for a woman experiencing severe preeclampsia? a.Risk for injury to the fetus related to uteroplacental insufficiency b.Risk for eclampsia c.Risk for deficient fluid volume related to increased sodium retention secondary to administration of MgSO4 d.Risk for increased cardiac output related to use of antihypertensive drugs

ANS: A Risk for injury to the fetus related to uteroplacental insufficiency is the most appropriate nursing diagnosis for this client scenario. Other diagnoses include Risk to fetus related to preterm birth and abruptio placentae. Eclampsia is a medical, not a nursing, diagnosis. There would be a risk for excess, not deficient, fluid volume related to increased sodium retention. There would be a risk for decreased, not increased, cardiac output related to the use of antihypertensive drugs.

Nurses should be aware of the differences experience can make in labor pain such as: a.Sensory pain for nulliparous women often is greater than for multiparous women during early labor. b.Affective pain for nulliparous women usually is less than for multiparous women throughout the first stage of labor. c.Women with a history of substance abuse experience more pain during labor. d.Multiparous women have more fatigue from labor and therefore experience more pain.

ANS: A Sensory pain is greater for nulliparous women because their reproductive tract structures are less supple. Affective pain is greater for nulliparous women during the first stage but decreases for both nulliparous and multiparous during the second stage. Women with a history of substance abuse experience the same amount of pain as those without such a history. Nulliparous women have longer labors and therefore experience more fatigue.

To help a woman reduce the severity of nausea caused by morning sickness, the nurse might suggest that she: a.Try a tart food or drink such as lemonade or salty foods such as potato chips. b.Drink plenty of fluids early in the day. c.Brush her teeth immediately after eating. d.Never snack before bedtime.

ANS: A Some women can tolerate tart or salty foods when they are nauseous. The woman should avoid drinking too much when nausea is most likely, but she should make up the fluid levels later in the day when she feels better. The woman should avoid brushing her teeth immediately after eating. A small snack of cereal and milk or yogurt before bedtime may help the stomach in the morning.

In assessing a woman for pain and discomfort management during labor, a nurse most likely would: a.Have the woman use a visual analog scale (VAS) to determine her level of pain. b.Note drowsiness as a sign that the medications were working. c.Interpret a womans fist clenching as an indication that she is angry at her male partner and the physician. d.Evaluate the womans skin turgor to see whether she needs a gentle oil massage.

ANS: A The VAS is a means of adding the womans assessment of her pain to the nurses observations. Drowsiness is a side effect of medications, not usually (sedatives aside) a sign of effectiveness. The fist clenching likely is a sign of apprehension that may need attention. Skin turgor, along with the moistness of the membranes and the concentration of the urine, is a sign that helps the nurse evaluate hydration.

A woman has chosen the calendar method of conception control. During the assessment process, it is most important that the nurse: a.Obtain a history of menstrual cycle lengths for the past 6 to 12 months. b.Determine the clients weight gain and loss pattern for the previous year. c.Examine skin pigmentation and hair texture for hormonal changes. d.Explore the clients previous experiences with conception control.

ANS: A The calendar method of conception control is based on the number of days in each cycle, counting from the first day of menses. The fertile period is determined after the lengths of menstrual cycles have been accurately recorded for 6 months. Weight gain or loss may be partly related to hormonal fluctuations, but it has no bearing on use of the calendar method. Integumentary changes may be related to hormonal changes, but they are not indicators for use of the calendar method. Exploring previous experiences with conception control may demonstrate client understanding and compliancy, but it is not the most important aspect to assess for discussion of the calendar method.

In caring for an immediate postpartum client, you note petechiae and oozing from her IV site. You would monitor her closely for the clotting disorder: a.Disseminated intravascular coagulation (DIC) b.Amniotic fluid embolism (AFE) c.Hemorrhage d.HELLP syndrome

ANS: A The diagnosis of DIC is made according to clinical findings and laboratory markers. Physical examination reveals unusual bleeding. Petechiae may appear around a blood pressure cuff on the womans arm. Excessive bleeding may occur from the site of slight trauma such as venipuncture sites. These symptoms are not associated with AFE, nor is AFE a bleeding disorder. Hemorrhage occurs for a variety of reasons in the postpartum client. These symptoms are associated with DIC. Hemorrhage would be a finding associated with DIC and is not a clotting disorder in and of itself. HELLP is not a clotting disorder, but it may contribute to the clotting disorder DIC.

A physician prescribes clomiphene citrate (Clomid, Serophene) for a woman experiencing infertility. She is very concerned about the risk of multiple births. The nurses most appropriate response is: a.This is a legitimate concern. Would you like to discuss this further before your treatment begins? b.No one has ever had more than triplets with Clomid. c.Ovulation will be monitored with ultrasound so that this will not happen. d.Ten percent is a very low risk, so you dont need to worry too much.

ANS: A The incidence of multiple pregnancies with the use of these medications is significantly increased. The patients concern is legitimate and should be discussed so that she can make an informed decision. Stating that no one has ever had more than triplets is inaccurate and negates the patients concerns. Ultrasound cannot ensure that a multiple pregnancy will not occur. The percentage quoted in this statement is inaccurate. The comment dont worry discredits the patients concern.

In relation to primary and secondary powers, the maternity nurse comprehends that: a.Primary powers are responsible for effacement and dilation of the cervix. b.Effacement generally is well ahead of dilation in women giving birth for the first time; they are closer together in subsequent pregnancies. c.Scarring of the cervix caused by a previous infection or surgery may make the delivery a bit more painful, but it should not slow or inhibit dilation. d.Pushing in the second stage of labor is more effective if the woman can breathe deeply and control some of her involuntary needs to push, as the nurse directs.

ANS: A The primary powers are responsible for dilation and effacement; secondary powers are concerned with expulsion of the fetus. Effacement generally is well ahead of dilation in first-timers; they are closer together in subsequent pregnancies. Scarring of the cervix may slow dilation. Pushing is more effective and less fatiguing when the woman begins to push only after she has the urge to do so.

Which contraceptive method has a failure rate of less than 25%? a.Standard days b.Periodic abstinence c.Postovulation d.Coitus interruptus

ANS: A The standard days variation on the calendar method has a failure rate of 12%. The periodic abstinence method has a failure rate of 25% or greater. The postovulation method has a failure rate of 25% or greater. The coitus interruptus method has a failure rate of 27% or greater.

To help clients manage discomfort and pain during labor, nurses should be aware that: a.The predominant pain of the first stage of labor is the visceral pain located in the lower portion of the abdomen. b.Referred pain is the extreme discomfort between contractions. c.The somatic pain of the second stage of labor is more generalized and related to fatigue. d.Pain during the third stage is a somewhat milder version of the second stage.

ANS: A This pain comes from cervical changes, distention of the lower uterine segment, and uterine ischemia. Referred pain occurs when the pain that originates in the uterus radiates to the abdominal wall, lumbosacral area of the back, iliac crests, and gluteal area. Second-stage labor pain is intense, sharp, burning, and localized. Third-stage labor pain is similar to that of the first stage.

Nurses should be aware that infertility: a.Is perceived differently by women and men. b.Has a relatively stable prevalence among the overall population and throughout a womans potential reproductive years. c.Is more likely the result of a physical flaw in the woman than in her male partner. d.Is the same thing as sterility.

ANS: A Women tend to be more stressed about infertility tests and to place more importance on having children. The prevalence of infertility is stable among the overall population, but it increases with a womans age, especially after age 40. Of cases with an identifiable cause, about 40% are related to female factors, 40% to male factors, and 20% to both partners. Sterility is the inability to conceive. Infertility, or subfertility, is a state of requiring a prolonged time to conceive.

A woman was treated recently for toxic shock syndrome (TSS). She has intercourse occasionally and uses over-the-counter protection. On the basis of her history, what contraceptive method should she and her partner avoid? a.Cervical cap b.Condom c.Vaginal film d.Vaginal sheath

ANS: A Women with a history of TSS should not use a cervical cap. Condoms, vaginal films, and vaginal sheaths are not contraindicated for a woman with a history of TSS.

39. Congenital anomalies can occur with the use of antiepileptic drugs (AEDs), including (Select all that apply): a.Cleft lip. b.Congenital heart disease. c.Neural tube defects. d.Gastroschisis. e.Diaphragmatic hernia

ANS: A, B, C Congenital anomalies that can occur with AEDs include cleft lip or palate, congenital heart disease, urogenital defects, and neural tube defects. Gastroschisis and diaphragmatic hernia are not associated with the use of AEDs.

Most women with uncomplicated pregnancies can use the nurse as their primary source for nutritional information. The nurse or midwife should refer a client to a registered dietitian for in-depth nutritional counseling in the following situations (Select all that apply). a. Preexisting or gestational illness such as diabetes b. Ethnic or cultural food patterns c. Obesity d. Vegetarian diet e. Allergy to tree nuts

ANS: A, B, C, D The nurse should be especially aware that conditions such as diabetes can require in-depth dietary planning and evaluation. To prevent issues with hypoglycemia and hyperglycemia and an increased risk for perinatal morbidity and mortality, this patient would benefit from a referral to a dietitian. Consultation with a dietitian may ensure that cultural food beliefs are congruent with modern knowledge of fetal development and that adjustments can be made to ensure that all nutritional needs are met. The obese pregnant patient may be under the misapprehension that because of her excess weight little or no weight gain is necessary. According to the Institute of Medicine, a client with a body mass index in the obese range should gain at least 7 kg to ensure a healthy outcome. This patient may require in-depth counseling on optimal food choices. The vegetarian client needs to have her dietary intake carefully assessed to ensure that the optimal combination of amino acids and protein intake is achieved. Very strict vegetarians (vegans) who consume only plant products may also require vitamin B and mineral supplementation. A patient with a food allergy would not alter that component of her diet during pregnancy; therefore, no additional consultation is necessary.

With regard to nutritional needs during lactation, a maternity nurse should be aware that: a.The mothers intake of vitamin C, zinc, and protein now can be lower than during pregnancy. b.Caffeine consumed by the mother accumulates in the infant, who may be unusually active and wakeful. c.Critical iron and folic acid levels must be maintained. d.Lactating women can go back to their prepregnant calorie intake.

ANS: B A lactating woman needs to avoid consuming too much caffeine. Vitamin C, zinc, and protein levels need to be moderately higher during lactation than during pregnancy. The recommendations for iron and folic acid are lower during lactation. Lactating women should consume about 500 kcal more than their prepregnancy intake, at least 1800 kcal daily overall.

A woman has a history of drug use and is screened for hepatitis B during the first trimester. What is an appropriate action? a.Provide a low-protein diet. b.Offer the vaccine. c.Discuss the recommendation to bottle-feed her baby. d.Practice respiratory isolation.

ANS: B A person who has a history of high risk behaviors should be offered the hepatitis B vaccine. Care is supportive and includes bed rest and a high-protein, low-fat diet. The first trimester is too early to discuss feeding methods with a woman in the high risk category. Hepatitis B is transmitted through blood.

While developing an intrapartum care plan for the patient in early labor, it is important that the nurse recognize that psychosocial factors may influence a womans experience of pain. These include (Select all that apply): a. Culture. b. Anxiety and fear. c. Previous experiences with pain. d. Intervention of caregivers. e. Support systems.

ANS: A, B, C, E Culture: a womans sociocultural roots influence how she perceives, interprets, and responds to pain during childbirth. Some cultures encourage loud and vigorous expressions of pain, whereas others value self-control. The nurse should avoid praising some behaviors (stoicism) while belittling others (noisy expression). Anxiety and fear: extreme anxiety and fear magnify sensitivity to pain and impair a womans ability to tolerate it. Anxiety and fear increase muscle tension in the pelvic area, which counters the expulsive forces of uterine contractions and pushing efforts. Previous experiences with pain: fear and withdrawal are a natural response to pain during labor. Learning about these normal sensations ahead of time helps a woman suppress her natural reactions of fear regarding the impending birth. If a woman previously had a long and difficult labor, she is likely to be anxious. She may also have learned ways to cope and may use these skills to adapt to the present labor experience. Support systems: an anxious partner is less able to provide help and support to a woman during labor. A womans family and friends can be an important source of support if they convey realistic and positive information about labor and delivery. Although the intervention of caregivers may be necessary for the well-being of the woman and her fetus, some interventions add discomfort to the natural pain of labor (i.e., fetal monitor straps, intravenous lines).

Intrauterine growth restriction (IUGR) is associated with numerous pregnancy-related risk factors (Select all that apply). a.Poor nutrition b.Maternal collagen disease c.Gestational hypertension d.Premature rupture of membranes e.Smoking

ANS: A, B, C, E Poor nutrition, maternal collagen disease, gestational hypertension, and smoking all are risk factors associated with IUGR. Premature rupture of membranes is associated with preterm labor, not IUGR.

The reported incidence of ectopic pregnancy in the United States has risen steadily over the past 2 decades. Causes include the increase in STDs accompanied by tubal infection and damage. The popularity of contraceptive devices such as the IUD has also increased the risk for ectopic pregnancy. The nurse who suspects that a patient has early signs of ectopic pregnancy should be observing her for symptoms such as (Select all that apply): a.Pelvic pain b.Abdominal pain c.Unanticipated heavy bleeding d.Vaginal spotting or light bleeding e.Missed period

ANS: A, B, D, E A missed period or spotting can easily be mistaken by the patient as early signs of pregnancy. More subtle signs depend on exactly where the implantation occurs. The nurse must be thorough in her assessment because pain is not a normal symptom of early pregnancy. As the fallopian tube tears open and the embryo is expelled, the patient often exhibits severe pain accompanied by intraabdominal hemorrhage. This may progress to hypovolemic shock with minimal or even no external bleeding. In about half of women, shoulder and neck pain results from irritation of the diaphragm from the hemorrhage.

Diabetes refers to a group of metabolic diseases characterized by hyperglycemia resulting from defects in insulin action, insulin secretion, or both. Over time, diabetes causes significant changes in the microvascular and macrovascular circulations. These complications include: a.Atherosclerosis. b.Retinopathy. c.IUFD. d.Nephropathy. e.Neuropathy. f.Autonomcs neuropathy.

ANS: A, B, D, E These structural changes are most likely to affect a variety of systems, including the heart, eyes, kidneys, and nerves. Intrauterine fetal death (stillbirth) remains a major complication of diabetes in pregnancy; however, this is a fetal complication.

The class of drugs known as opioid analgesics (butorphanol, nalbuphine) is not suitable for administration to women with known opioid dependence. The antagonistic activity could precipitate withdrawal symptoms (abstinence syndrome) in both mothers and newborns. Signs of opioid/narcotic withdrawal in the mother would include (Select all that apply): a. Yawning, runny nose. b. Increase in appetite. c. Chills and hot flashes. d. Constipation. e. Irritability, restlessness.

ANS: A, C, E The woman experiencing maternal opioid withdrawal syndrome will exhibit yawning, runny nose, sneezing, anorexia, chills or hot flashes, vomiting, diarrhea, abdominal pain, irritability, restlessness, muscle spasms, weakness, and drowsiness. It is important for the nurse to assess both mother and baby and to plan care accordingly.

A client who has undergone a dilation and curettage for early pregnancy loss is likely to be discharged the same day. The nurse must ensure that vital signs are stable, bleeding has been controlled, and the woman has adequately recovered from the administration of anesthesia. To promote an optimal recovery, discharge teaching should include (Select all that apply): a.Iron supplementation. b.Resumption of intercourse at 6 weeks following the procedure. c.Referral to a support group if necessary. d.Expectation of heavy bleeding for at least 2 weeks. e.Emphasizing the need for rest.

ANS: A, C, E The woman should be advised to consume a diet high in iron and protein. For many women iron supplementation also is necessary. Acknowledge that the client has experienced a loss, albeit early. She can be taught to expect mood swings and possibly depression. Referral to a support group, clergy, or professional counseling may be necessary. Discharge teaching should emphasize the need for rest. Nothing should be placed in the vagina for 2 weeks after the procedure. This includes tampons and vaginal intercourse. The purpose of this recommendation is to prevent infection. Should infection occur, antibiotics may be prescribed. The client should expect a scant, dark discharge for 1 to 2 weeks. Should heavy, profuse, or bright bleeding occur, she should be instructed to contact her provider.

You (the nurse) are reviewing the educational packet provided to a client about tubal ligation. What is an important fact you should point out (Select all that apply)? a.It is highly unlikely that you will become pregnant after the procedure. b.This is an effective form of 100% permanent sterilization. You wont be able to get pregnant. c.Sterilization offers some form of protection against sexually transmitted infections (STIs). d.Sterilization offers no protection against STIs. e.Your menstrual cycle will greatly increase after your sterilization.

ANS: A, D A woman is unlikely to become pregnant after tubal ligation, although it is not 100% effective. Sterilization offers no protection against STIs. The menstrual cycle typically remains the same after a tubal ligation.

In teaching the pregnant adolescent about nutrition, the nurse should: a.Emphasize the need to eliminate common teen snack foods because they are too high in fat and sodium. b.Determine the weight gain needed to meet adolescent growth and add 35 lb. c.Suggest that she not eat at fast-food restaurants to avoid foods of poor nutritional value. d.Realize that most adolescents are unwilling to make dietary changes during pregnancy.

ANS: B Adolescents should gain in the upper range of the recommended weight gain. They also need to gain weight that would be expected for their own normal growth. Changes in the diet should be kept at a minimum. Snack foods can be included in moderation, and other foods can be added to make up for the lost nutrients. Eliminating fast foods would make the adolescent appear different to her peers. The patient should be taught to choose foods that add needed nutrients. Adolescents are willing to make changes; however, they still have the need to be similar to their peers.

Concerning the use and abuse of legal drugs or substances, nurses should be aware that: a.Although cigarette smoking causes a number of health problems, it has little direct effect on maternity-related health. b.Caucasian women are more likely to experience alcohol-related problems. c.Coffee is a stimulant that can interrupt body functions and has been related to birth defects. d.Prescription psychotherapeutic drugs taken by the mother do not affect the fetus; otherwise, they would not have been prescribed.

ANS: B African-American and poor women are more likely to use illicit substances, particularly cocaine, whereas Caucasian and educated women are more likely to use alcohol. Cigarette smoking impairs fertility and is a cause of low birth weight. Caffeine consumption has not been related to birth defects. Psychotherapeutic drugs have some effect on the fetus, and that risk must be weighed against their benefit to the mother.

With regard to a pregnant womans anxiety and pain experience, nurses should be aware that: a.Even mild anxiety must be treated. b.Severe anxiety increases tension, which increases pain, which in turn increases fear and anxiety, and so on. c.Anxiety may increase the perception of pain, but it does not affect the mechanism of labor. d.Women who have had a painful labor will have learned from the experience and have less anxiety the second time because of increased familiarity.

ANS: B Anxiety and pain reinforce each other in a negative cycle. Mild anxiety is normal for a woman in labor and likely needs no special treatment other than the standard reassurances. Anxiety increases muscle tension and ultimately can build sufficiently to slow the progress of labor. Unfortunately, an anxious, painful first labor is likely to carry over, through expectations and memories, into an anxious and painful experience in the second pregnancy

Three servings of milk, yogurt, or cheese plus two servings of meat, poultry, or fish adequately supply the recommended amount of protein for a pregnant woman. Many patients are concerned about the increased levels of mercury in fish and may be afraid to include this source of nutrients in their diet. Sound advice by the nurse to assist the client in determining which fish is safe to consume would include: a.Canned white tuna is a preferred choice. b.Avoid shark, swordfish, and mackerel. c.Fish caught in local waterways are the safest. d.Salmon and shrimp contain high levels of mercury.

ANS: B As a precaution, the pregnant patient should avoid eating all of these and the less common tilefish. High levels of mercury can harm the developing nervous system of the fetus. It is essential for the nurse to assist the client in understanding the differences between numerous sources of this product. A pregnant client can 12 ounces a week of canned light tuna; however, canned white, albacore, or tuna steaks contain higher levels of mercury and should be limited to no more than 6 ounces per week. It is a common misconception that fish caught in local waterways are the safest. Pregnant women and mothers of young children should check with local advisories about the safety of fish caught by families and friends in nearby bodies of water. If no information is available, these fish sources should be avoided, limited to less than 6 ounces, or the only fish consumed that week. Commercially caught fish that are low in mercury include salmon, shrimp, pollock, or catfish.

After change-of-shift report the nurse assumes care of a multiparous client in labor. The woman is complaining of pain that radiates to her abdominal wall, lower back, and buttocks and down her thighs. Before implementing a plan of care, the nurse should understand that this type of pain is: a.Visceral. b.Referred. c.Somatic. d.Afterpain.

ANS: B As labor progresses the woman often experiences referred pain. This occurs when pain that originates in the uterus radiates to the abdominal wall, the lumbosacral area of the back, the gluteal area, and thighs. The woman usually has pain only during a contraction and is free from pain between contractions. Visceral pain is that which predominates in the first stage of labor. This pain originates from cervical changes, distention of the lower uterine segment, and uterine ischemia. Visceral pain is located over the lower portion of the abdomen. Somatic pain is described as intense, sharp, burning, and well localized. This results from stretching of the perineal tissues and the pelvic floor. This occurs during the second stage of labor. Pain experienced during the third stage of labor or afterward during the early postpartum period is uterine. This pain is very similar to that experienced in the first stage of labor.

Which contraceptive method best protects against sexually transmitted infections (STIs) and human immunodeficiency virus (HIV)? a.Periodic abstinence b.Barrier methods c.Hormonal methods d.They all offer about the same protection.

ANS: B Barrier methods such as condoms best protect against STIs and HIV. Periodic abstinence and hormonal methods (the pill) offer no protection against STIs or HIV.

It is important for the nurse to develop a realistic birth plan with the pregnant woman in her care. The nurse can explain that a major advantage of nonpharmacologic pain management is: a.Greater and more complete pain relief is possible. b.No side effects or risks to the fetus are involved. c.The woman remains fully alert at all times. d.A more rapid labor is likely.

ANS: B Because nonpharmacologic pain management does not include analgesics, adjunct drugs, or anesthesia, it is harmless to the mother and the fetus. There is less pain relief with nonpharmacologic pain management during childbirth. The womans alertness is not altered by medication; however, the increase in pain will decrease alertness. Pain management may or may not alter the length of labor. At times when pain is decreased, the mother relaxes and labor progresses at a quicker pace.

A woman who has a seizure disorder and takes barbiturates and phenytoin sodium daily asks the nurse about the pill as a contraceptive choice. The nurses most appropriate response would be: a.This is a highly effective method, but it has some side effects. b.Your current medications will reduce the effectiveness of the pill. c.The pill will reduce the effectiveness of your seizure medication. d.This is a good choice for a woman of your age and personal history.

ANS: B Because the liver metabolizes oral contraceptives, their effectiveness is reduced when they are taken simultaneously with anticonvulsants. The statement Your current medications will reduce the effectiveness of the pill is true, but it is not the most appropriate response. The anticonvulsant will reduce the effectiveness of the pill, not the other way around. The statement This is a good choice for a woman of your age and personal history does not teach the client that the effectiveness of the pill may be reduced because of her anticonvulsant therapy.

While providing care in an obstetric setting, the nurse should understand that postpartum care of the woman with cardiac disease: a.Is the same as that for any pregnant woman. b.Includes rest, stool softeners, and monitoring of the effect of activity. c.Includes ambulating frequently, alternating with active range of motion. d.Includes limiting visits with the infant to once per day.

ANS: B Bed rest may be ordered, with or without bathroom privileges. Bowel movements without stress or strain for the woman are promoted with stool softeners, diet, and fluid. Care of the woman with cardiac disease in the postpartum period is tailored to the womans functional capacity. The woman will be on bed rest to conserve energy and reduce the strain on the heart. Although the woman may need help caring for the infant, breastfeeding and infant visits are not contraindicated.

An 18-year-old client who has reached 16 weeks of gestation was recently diagnosed with pregestational diabetes. She attends her centering appointment accompanied by one of her girlfriends. This young woman appears more concerned about how her pregnancy will affect her social life than about her recent diagnosis of diabetes. Several nursing diagnoses are applicable to assist in planning adequate care. The most appropriate diagnosis at this time is: a.Risk for injury to the fetus related to birth trauma. b.Noncompliance related to lack of understanding of diabetes and pregnancy and requirements of the treatment plan. c.Deficient knowledge related to insulin administration. d.Risk for injury to the mother related to hypoglycemia or hyperglycemia.

ANS: B Before a treatment plan is developed or goals for the outcome of care are outlined, this client must come to an understanding of diabetes and the potential effects on her pregnancy. She appears to have greater concern for changes to her social life than adoption of a new self-care regimen. Risk for injury to the fetus related to either placental insufficiency or birth trauma may come much later in the pregnancy. At this time the client is having difficulty acknowledging the adjustments that she needs to make to her lifestyle to care for herself during pregnancy. The client may not yet be on insulin. Insulin requirements increase with gestation. The importance of glycemic control must be part of health teaching for this client. However, she has not yet acknowledged that changes to her lifestyle need to be made, and she may not participate in the plan of care until understanding takes place.

Women with an inadequate weight gain during pregnancy are at higher risk of giving birth to an infant with: a.Spina bifida. b.Intrauterine growth restriction. c.Diabetes mellitus. d.Down syndrome.

ANS: B Both normal-weight and underweight women with inadequate weight gain have an increased risk of giving birth to an infant with intrauterine growth restriction. Spina bifida, diabetes mellitus, and Down syndrome are not associated with inadequate maternal weight gain.

Maternal phenylketonuria (PKU) is an important health concern during pregnancy because: a.It is a recognized cause of preterm labor. b.The fetus may develop neurologic problems. c.A pregnant woman is more likely to die without dietary control. d.Women with PKU are usually retarded and should not reproduce.

ANS: B Children born to women with untreated PKU are more likely to be born with mental retardation, microcephaly, congenital heart disease, and low birth weight. Maternal PKU has no effect on labor. Women without dietary control of PKU are more likely to miscarry or bear a child with congenital anomalies. Screening for undiagnosed maternal PKU at the first prenatal visit may be warranted, especially in individuals with a family history of the disorder, with low intelligence of uncertain etiology, or who have given birth to microcephalic infants.

Nurses caring for antepartum women with cardiac conditions should be aware that: a.Stress on the heart is greatest in the first trimester and the last 2 weeks before labor. b.Women with class II cardiac disease should avoid heavy exertion and any activity that causes even minor symptoms. c.Women with class III cardiac disease should have 8 to 10 hours of sleep every day and limit housework, shopping, and exercise. d.Women with class I cardiac disease need bed rest through most of the pregnancy and face the possibility of hospitalization near term.

ANS: B Class II cardiac disease is symptomatic with ordinary activity. Women in this category need to avoid heavy exertion and limit regular activities as symptoms dictate. Stress is greatest between weeks 28 and 32, when homodynamic changes reach their maximum. Class III cardiac disease is symptomatic with less than ordinary activity. These women need bed rest most of the day and face the possibility of hospitalization near term. Class I cardiac disease is asymptomatic at normal levels of activity. These women can carry on limited normal activities with discretion, although they still need a good amount of sleep.

To prevent gastrointestinal upset, clients should be instructed to take iron supplements: a.On a full stomach. b.At bedtime. c.After eating a meal. d.With milk.

ANS: B Clients should be instructed to take iron supplements at bedtime. Iron supplements are best absorbed if they are taken when the stomach is empty. Bran, tea, coffee, milk, and eggs may reduce absorption. Iron can be taken at bedtime if abdominal discomfort occurs when it is taken between meals.

Diabetes in pregnancy puts the fetus at risk in several ways. Nurses should be aware that: a.With good control of maternal glucose levels, sudden and unexplained stillbirth is no longer a major concern. b.The most important cause of perinatal loss in diabetic pregnancy is congenital malformations. c.Infants of mothers with diabetes have the same risks for respiratory distress syndrome because of the careful monitoring. d.At birth the neonate of a diabetic mother is no longer in any risk.

ANS: B Congenital malformations account for 30% to 50% of perinatal deaths. Even with good control, sudden and unexplained stillbirth remains a major concern. Infants of diabetic mothers are at increased risk for respiratory distress syndrome. The transition to extrauterine life often is marked by hypoglycemia and other metabolic abnormalities.

An unmarried young woman describes her sex life as active and involving many partners. She wants a contraceptive method that is reliable and does not interfere with sex. She requests an intrauterine device (IUD). The nurses most appropriate response is: a.The IUD does not interfere with sex. b.The risk of pelvic inflammatory disease (PID) will be higher for you. c.The IUD will protect you from sexually transmitted infections (STIs). d.Pregnancy rates are high with IUDs.

ANS: B Disadvantages of IUDs include an increased risk of PID in the first 20 days after insertion and the risks of bacterial vaginosis and uterine perforation. The IUD offers no protection against STIs or human immunodeficiency virus. Because this woman has multiple sex partners, she is at higher risk of developing a STI. The IUD does not protect against infection, as does a barrier method. Although the statement The IUD does not interfere with sex may be correct, it is not the most appropriate response. The IUD offers no protection from STIs. The typical failure rate of the IUD in the first year of use is 0.8%.

In the first trimester, ultrasonography can be used to gain information on: a.Amniotic fluid volume. b.Location of Gestational sacs c.Placental location and maturity. d.Cervical length.

ANS: B During the first trimester, ultrasound examination is performed to obtain information regarding the number, size, and location of gestatials sacs; the presence or absence of fetal cardiac and body movements; the presences or absence of uterine abnormalities (e.g., bicornuate uterus or fibroids) or adnexal masses (e.g., ovarian cysts or an ectopic pregnancy); and pregnancy dating.

Which statement is the best rationale for assessing maternal vital signs between contractions? a.During a contraction, assessing fetal heart rates is the priority. b.Maternal circulating blood volume increases temporarily during contractions. c.Maternal blood flow to the heart is reduced during contractions. d.Vital signs taken during contractions are not accurate.

ANS: B During uterine contractions, blood flow to the placenta temporarily stops, causing a relative increase in the mothers blood volume, which in turn temporarily increases blood pressure and slows pulse. It is important to monitor fetal response to contractions; however, this question is concerned with the maternal vital signs. Maternal blood flow is increased during a contraction. Vital signs are altered by contractions but are considered accurate for that period of time

A pregnant woman experiencing nausea and vomiting should: a.Drink a glass of water with a fat-free carbohydrate before getting out of bed in the morning. b.Eat small, frequent meals (every 2 to 3 hours). c.Increase her intake of high-fat foods to keep the stomach full and coated. d.Limit fluid intake throughout the day.

ANS: B Eating small, frequent meals is the correct suggestion for a woman experiencing nausea and vomiting. A pregnant woman experiencing nausea and vomiting should avoid consuming fluids early in the day or when nauseated, but should compensate by drinking fluids at other times. A pregnant woman experiencing nausea and vomiting should reduce her intake of fried and other fatty foods.

With regard to systemic analgesics administered during labor, nurses should be aware that: a.Systemic analgesics cross the maternal blood-brain barrier as easily as they do the fetal blood-brain barrier. b.Effects on the fetus and newborn can include decreased alertness and delayed sucking. c.Intramuscular administration (IM) is preferred over intravenous (IV) administration. d.IV patient-controlled analgesia (PCA) results in increased use of an analgesic

ANS: B Effects depend on the specific drug given, the dosage, and the timing. Systemic analgesics cross the fetal blood-brain barrier more readily than the maternal blood-brain barrier. IV administration is preferred over IM administration because the drug acts faster and more predictably. PCA results in decreased use of an analgesic.

Which test used to diagnose the basis of infertility is done during the luteal or secretory phase of the menstrual cycle? a.Hysterosalpingogram b.Endometrial biopsy c.Laparoscopy d.Follicle-stimulating hormone (FSH) level

ANS: B Endometrial biopsy is scheduled after ovulation, during the luteal phase of the menstrual cycle. A hysterosalpingogram is scheduled 2 to 5 days after menstruation to avoid flushing potentially fertilized ovum out through a uterine tube into the peritoneal cavity. Laparoscopy usually is scheduled early in the menstrual cycle. Hormone analysis is performed to assess endocrine function of the hypothalamic-pituitary-ovarian axis when menstrual cycles are absent or irregular.

A married couple is discussing alternatives for pregnancy prevention and has asked about fertility awareness methods (FAMs). The nurses most appropriate reply is: a.Theyre not very effective, and its very likely youll get pregnant. b.They can be effective for many couples, but they require motivation. c.These methods have a few advantages and several health risks. d.You would be much safer going on the pill and not having to worry.

ANS: B FAMs are effective with proper vigilance about ovulatory changes in the body and adherence to coitus intervals. They are effective if used correctly by a woman with a regular menstrual cycle. The typical failure rate for all FAMs is 25% during the first year of use. FAMs have no associated health risks. The use of birth control has associated health risks. In addition, taking a pill daily requires compliance on the clients part.

With regard to the assessment of female, male, and couple infertility, nurses should be aware that: a.The couples religious, cultural, and ethnic backgrounds provide emotional clutter that does not affect the clinical scientific diagnosis. b.The investigation takes 3 to 4 months and a significant financial investment. c.The woman is assessed first; if she is not the problem, the male partner is analyzed. d.Semen analysis is for men; the postcoital test is for women.

ANS: B Fertility assessment and diagnosis take time, money, and commitment from the couple. Religious, cultural, and ethnic-bred attitudes about fertility and related issues always have an impact on diagnosis and assessment. Both partners are assessed systematically and simultaneously, as individuals and as a couple. Semen analysis is for men, but the postcoital test is for the couple.

The slight overlapping of cranial bones or shaping of the fetal head during labor is called: a.Lightening. b.Molding. c.Ferguson reflex. d.Valsalva maneuver.

ANS: B Fetal head formation is called molding. Molding also permits adaptation to various diameters of the maternal pelvis. Lightening is the mothers sensation of decreased abdominal distention, which usually occurs the week before labor. The Ferguson reflex is the contraction urge of the uterus after stimulation of the cervix. The Valsalva maneuver describes conscious pushing during the second stage of labor.

A woman inquires about herbal alternative methods for improving fertility. Which statement by the nurse is the most appropriate when instructing the client in which herbal preparations to avoid while trying to conceive? a.You should avoid nettle leaf, dong quai, and vitamin E while you are trying to get pregnant. b.You may want to avoid licorice root, lavender, fennel, sage, and thyme while you are trying to conceive. c.You should not take anything with vitamin E, calcium, or magnesium. They will make you infertile. d.Herbs have no bearing on fertility.

ANS: B Herbs that a woman should avoid while trying to conceive include licorice root, yarrow, wormwood, ephedra, fennel, golden seal, lavender, juniper, flaxseed, pennyroyal, passionflower, wild cherry, cascara, sage, thyme, and periwinkle. Nettle leaf, dong quai, and vitamin E all promote fertility. Vitamin E, calcium, and magnesium may promote fertility and conception. All supplements and herbs should be purchased from trusted sources.

Maternal serum alpha-fetoprotein (MSAFP) screening indicates an elevated level. MSAFP screening is repeated and again is reported as higher than normal. What would be the next step in the assessment sequence to determine the well-being of the fetus? a.Percutaneous umbilical blood sampling (PUBS) b.Ultrasound for fetal anomalies c.Biophysical profile (BPP) for fetal well-being d.Amniocentesis for genetic anomalies

ANS: B If MSAFP findings are abnormal, follow-up procedures include genetic counseling for families with a history of neural tube defect, repeated MSAFP screening, ultrasound examination, and possibly amniocentesis. Indications for use of PUBS include prenatal diagnosis of inherited blood disorders, karyotyping of malformed fetuses, detection of fetal infection, determination of the acid-base status of fetuses with intrauterine growth restriction, and assessment and treatment of isoimmunization and thrombocytopenia in the fetus. BPP is a method of assessing fetal well-being in the third trimester. Before amniocentesis is considered, the client first would have an ultrasound for direct visualization of the fetus.

The primary difference between the labor of a nullipara and that of a multipara is the: a.Amount of cervical dilation. b.Total duration of labor. c.Level of pain experienced. d.Sequence of labor mechanisms

ANS: B In a first-time pregnancy, descent is usually slow but steady; in subsequent pregnancies, descent is more rapid, resulting in a shorter duration of labor. Cervical dilation is the same for all labors. Level of pain is individual to the woman, not to the number of labors she has experienced. The sequence of labor mechanisms is the same with all labors

Which maternal condition always necessitates delivery by cesarean section? a.Partial abruptio placentae b.Total placenta previa c.Ectopic pregnancy d.Eclampsia

ANS: B In total placenta previa, the placenta completely covers the cervical os. The fetus would die if a vaginal delivery occurred. If the mother has stable vital signs and the fetus is alive, a vaginal delivery can be attempted in cases of partial abruptio placentae. If the fetus has died, a vaginal delivery is preferred. The most common ectopic pregnancy is a tubal pregnancy, which is usually detected and treated in the first trimester. Labor can be safely induced if the eclampsia is under control.

The nurse caring for a woman hospitalized for hyperemesis gravidarum should expect that initial treatment to involve: a.Corticosteroids to reduce inflammation. b.IV therapy to correct fluid and electrolyte imbalances. c.An antiemetic, such as pyridoxine, to control nausea and vomiting. d.Enteral nutrition to correct nutritional deficits.

ANS: B Initially, the woman who is unable to keep down clear liquids by mouth requires IV therapy for correction of fluid and electrolyte imbalances. Corticosteroids have been used successfully to treat refractory hyperemesis gravidarum; however, they are not the expected initial treatment for this disorder. Pyridoxine is vitamin B6, not an antiemetic. Promethazine, a common antiemetic, may be prescribed. In severe cases of hyperemesis gravidarum, enteral nutrition via a feeding tube may be necessary to correct maternal nutritional deprivation. This is not an initial treatment for this patient.

The most important reason for evaluating the pattern of weight gain in pregnancy is to: a.Prevent excessive adipose tissue deposits b.Identify potential nutritional problems or complications of pregnancy c.Assess the need to limit caloric intake in obese women d.Determine cultural influences on the womans diet

ANS: B Maternal and fetal risks in pregnancy are increased when the mother is significantly overweight. Excessive adipose tissue may occur with excess weight gain; however, this is not the reason for monitoring the weight gain pattern. It is important to monitor the pattern of weight gain to identify complications. The pattern of weight gain is not influenced by cultural influences.

The use of methamphetamine (meth) has been described as a significant drug problem in the United States. In order to provide adequate nursing care to this client population the nurse must be cognizant that methamphetamine: a.Is similar to opiates. b.Is a stimulant with vasoconstrictive characteristics. c.Should not be discontinued during pregnancy. d.Is associated with a low rate of relapse.

ANS: B Methamphetamines are stimulants with vasoconstrictive characteristics similar to cocaine and are used similarly. As is the case with cocaine users, methamphetamine users are urged to immediately stop all use during pregnancy. Unfortunately, because methamphetamine users are extremely psychologically addicted, the rate of relapse is very high.

An abortion in which the fetus dies but is retained within the uterus is called a(n): a.Inevitable abortion b.Missed abortion c.Incomplete abortion d.Threatened abortion

ANS: B Missed abortion refers to retention of a dead fetus in the uterus. An inevitable abortion means that the cervix is dilating with the contractions. An incomplete abortion means that not all of the products of conception were expelled. With a threatened abortion the woman has cramping and bleeding but not cervical dilation.

The role of the nurse with regard to informed consent is to: a.Inform the client about the procedure and have her sign the consent form. b.Act as a client advocate and help clarify the procedure and the options. c.Call the physician to see the client. d.Witness the signing of the consent form.

ANS: B Nurses play a part in the informed consent process by clarifying and describing procedures or by acting as the womans advocate and asking the primary health care provider for further explanations. The physician is responsible for informing the woman of her options, explaining the procedure, and advising the client about potential risk factors. The physician must be present to explain the procedure to the client. However, the nurses responsibilities go further than simply asking the physician to see the client. The nurse may witness the signing of the consent form. However, depending on the states guidelines, the womans husband or another hospital health care employee may sign as witness.

Preconception counseling is critical to the outcome of diabetic pregnancies because poor glycemic control before and during early pregnancy is associated with: a.Frequent episodes of maternal hypoglycemia. b.Congenital anomalies in the fetus. c.Polyhydramnios. d.Hyperemesis gravidarum.

ANS: B Preconception counseling is particularly important because strict metabolic control before conception and in the early weeks of gestation is instrumental in decreasing the risks of congenital anomalies. Frequent episodes of maternal hypoglycemia may occur during the first trimester (not before conception) as a result of hormone changes and the effects on insulin production and usage. Hydramnios occurs about 10 times more often in diabetic pregnancies than in nondiabetic pregnancies. Typically it is seen in the third trimester of pregnancy. Hyperemesis gravidarum may exacerbate hypoglycemic events because the decreased food intake by the mother and glucose transfer to the fetus contribute to hypoglycemia.

A 41-week pregnant multigravida presents in the labor and delivery unit after a nonstress test indicated that her fetus could be experiencing some difficulties in utero. Which diagnostic tool would yield more detailed information about the fetus? a.Ultrasound for fetal anomalies b.Biophysical profile (BPP) c.Maternal serum alpha-fetoprotein (MSAFP) screening d.Percutaneous umbilical blood sampling (PUBS)

ANS: B Real-time ultrasound permits detailed assessment of the physical and physiologic characteristics of the developing fetus and cataloging of normal and abnormal biophysical responses to stimuli. BPP is a noninvasive, dynamic assessment of a fetus that is based on acute and chronic markers of fetal disease. An ultrasound for fetal anomalies would most likely have been performed earlier in the pregnancy. It is too late in the pregnancy to perform MSAFP screening. Also, MSAFP screening does not provide information related to fetal well-being. Indications for PUBS include prenatal diagnosis or inherited blood disorders, karyotyping of malformed fetuses, detection of fetal infection, determination of the acid-base status of a fetus with IUGR, and assessment and treatment of isoimmunization and thrombocytopenia in the fetus.

A first-time mother is concerned about the type of medications she will receive during labor. She is in a fair amount of pain and is nauseous. In addition, she appears to be very anxious. You explain that opioid analgesics often are used with sedatives because: a.The two together work the best for you and your baby. b.Sedatives help the opioid work better, and they also will assist you to relax and relieve your nausea. c.They work better together so you can sleep until you have the baby. d.This is what the doctor has ordered for you.

ANS: B Sedatives can be used to reduce the nausea and vomiting that often accompany opioid use. In addition, some ataractics reduce anxiety and apprehension and potentiate the opioid analgesic affects. A potentiator may cause the two drugs to work together more effectively, but it does not ensure maternal or fetal complications will not occur. Sedation may be a related effect of some ataractics, but it is not the goal. Furthermore, a woman is unlikely to be able to sleep through transitional labor and birth. This is what the doctor has ordered for you may be true, but it is not an acceptable comment for the nurse to make.

The nurse should be aware that an effective plan to achieve adequate pain relief without maternal risk is most effective if: a.The mother gives birth without any analgesic or anesthetic. b.The mother and familys priorities and preferences are incorporated into the plan. c.The primary health care provider decides the best pain relief for the mother and family. d.The nurse informs the family of all alternative methods of pain relief available in the hospital setting.

ANS: B The assessment of the woman, her fetus, and her labor is a joint effort of the nurse and the primary health care providers, who consult with the woman about their findings and recommendations. The needs of each woman are different, and many factors must be considered before a decision is made whether pharmacologic methods, nonpharmacologic methods, or a combination of the two will be used to manage labor pain.

To teach patients about the process of labor adequately, the nurse knows that which event is the best indicator of true labor? a.Bloody show b.Cervical dilation and effacement c.Fetal descent into the pelvic inlet d.Uterine contractions every 7 minutes

ANS: B The conclusive distinction between true and false labor is that contractions of true labor cause progressive change in the cervix. Bloody show can occur before true labor. Fetal descent can occur before true labor. False labor may have contractions that occur this frequently; however, this is usually inconsistent.

The nurse has received report regarding her patient in labor. The womans last vaginal examination was recorded as 3 cm, 30%, and ?2-2. The nurses interpretation of this assessment is that: a.The cervix is effaced 3 cm, it is dilated 30%, and the presenting part is 2 cm above the ischial spines. b.The cervix is 3 cm dilated, it is effaced 30%, and the presenting part is 2 cm above the ischial spines. c.The cervix is effaced 3 cm, it is dilated 30%, and the presenting part is 2 cm below the ischial spines. d.The cervix is dilated 3 cm, it is effaced 30%, and the presenting part is 2 cm below the ischial spines.

ANS: B The correct description of the vaginal examination for this woman in labor is the cervix is 3 cm dilated, it is effaced 30%, and the presenting part is 2 cm above the ischial spines. The sterile vaginal examination is recorded as centimeters of cervical dilation, percentage of cervical dilation, and the relationship of the presenting part to the ischial spines (either above or below).

A woman has been diagnosed with a high risk pregnancy. She and her husband come into the office in a very anxious state. She seems to be coping by withdrawing from the discussion, showing declining interest. The nurse can best help the couple by: a.Telling her that the physician will isolate the problem with more tests. b.Encouraging her and urging her to continue with childbirth classes. c.Becoming assertive and laying out the decisions the couple needs to make. d.Downplaying her risks by citing success rate studies.

ANS: B The nurse can best help the woman and her husband regain a sense of control in their lives by providing support and encouragement (including active involvement in preparations and classes). The nurse can try to present opportunities for the couple to make as many choices as possible in prenatal care.

Magnesium sulfate is given to women with preeclampsia and eclampsia to: a.Improve patellar reflexes and increase respiratory efficiency. b.Shorten the duration of labor. c.Prevent and treat convulsions. d.Prevent a boggy uterus and lessen lochial flow.

ANS: C Magnesium sulfate is the drug of choice to prevent convulsions, although it can generate other problems. Loss of patellar reflexes and respiratory depression are signs of magnesium toxicity. Magnesium sulfate can increase the duration of labor. Women are at risk for a boggy uterus and heavy lochial flow as a result of magnesium sulfate therapy.

26. A 26-year-old pregnant woman, gravida 2, para 1-0-0-1 is 28 weeks pregnant when she experiences bright red, painless vaginal bleeding. On her arrival at the hospital, what would be an expected diagnostic procedure? a.Amniocentesis for fetal lung maturity b.Ultrasound for placental location c.Contraction stress test (CST) d.Internal fetal monitoring

ANS: B The presence of painless bleeding should always alert the health care team to the possibility of placenta previa. This can be confirmed through ultrasonography. Amniocentesis would not be performed on a woman who is experiencing bleeding. In the event of an imminent delivery, the fetus would be presumed to have immature lungs at this gestational age, and the mother would be given corticosteroids to aid in fetal lung maturity. A CST would not be performed at a preterm gestational age. Furthermore, bleeding would be a contraindication to this test. Internal fetal monitoring would be contraindicated in the presence of bleeding.

A male client asks the nurse why it is better to purchase condoms that are not lubricated with nonoxynol-9 (a common spermicide). The nurses most appropriate response is: a.The lubricant prevents vaginal irritation. b.Nonoxynol-9 does not provide protection against sexually transmitted infections, as originally thought; it has also been linked to an increase in the transmission of human immunodeficiency virus and can cause genital lesions. c.The additional lubrication improves sex. d.Nonoxynol-9 improves penile sensitivity.

ANS: B The statement Nonoxynol-9 does not provide protection against sexually transmitted infections, as originally thought; it has also been linked to an increase in the transmission of human immunodeficiency virus and can cause genital lesions is true. Nonoxynol-9 may cause vaginal irritation, has no effect on the quality of sexual activity, and has no effect on penile sensitivity.

As relates to fetal positioning during labor, nurses should be aware that: a.Position is a measure of the degree of descent of the presenting part of the fetus through the birth canal. b.Birth is imminent when the presenting part is at +4 to +5 cm below the spine. c.The largest transverse diameter of the presenting part is the suboccipitobregmatic diameter. d.Engagement is the term used to describe the beginning of labor.

ANS: B The station of the presenting part should be noted at the beginning of labor so that the rate of descent can be determined. Position is the relation of the presenting part of the fetus to the four quadrants of the mothers pelvis;station is the measure of degree of descent. The largest diameter usually is the biparietal diameter. The suboccipitobregmatic diameter is the smallest, although one of the most critical. Engagement often occurs in the weeks just before labor in nulliparas and before or during labor in multiparas.

With regard to the use of intrauterine devices (IUDs), nurses should be aware that: a.Return to fertility can take several weeks after the device is removed. b.IUDs containing copper can provide an emergency contraception option if inserted within a few days of unprotected intercourse. c.IUDs offer the same protection against sexually transmitted infections (STIs) as the diaphragm. d.Consent forms are not needed for IUD insertion.

ANS: B The woman has up to 8 days to insert the IUD after unprotected sex. Return to fertility is immediate after removal of the IUD. IUDs offer no protection for STIs. A consent form is required for insertion, as is a negative pregnancy test.

If an opioid antagonist is administered to a laboring woman, she should be told that: a.Her pain will decrease. b.Her pain will return. c.She will feel less anxious. d.She will no longer feel the urge to push.

ANS: B The woman should be told that the pain that was relieved by the opioid analgesic will return with administration of the opioid antagonist. Opioid antagonists, such as Narcan, promptly reverse the central nervous system (CNS) depressant effects of opioids. In addition, the antagonist counters the effect of the stress-induced levels of endorphins. An opioid antagonist is especially valuable if labor is more rapid than expected and birth is anticipated when the opioid is at its peak effect

A 26-year-old primigravida has come to the clinic for her regular prenatal visit at 12 weeks. She appears thin and somewhat nervous. She reports that she eats a well-balanced diet, although her weight is 5 pounds less than it was at her last visit. The results of laboratory studies confirm that she has a hyperthyroid condition. Based on the available data, the nurse formulates a plan of care. What nursing diagnosis is most appropriate for the woman at this time? a.Deficient fluid volume b.Imbalanced nutrition: less than body requirements c.Imbalanced nutrition: more than body requirements d.Disturbed sleep pattern

ANS: B This clients clinical cues include weight loss, which would support the nursing diagnosis of Imbalanced nutrition: less than body requirements. No clinical signs or symptoms support the nursing diagnosis of Deficient fluid volume. This client reports weight loss, not weight gain. Imbalanced nutrition: more than body requirements is not an appropriate nursing diagnosis. Although the client reports nervousness, based on the clients other clinical symptoms the most appropriate nursing diagnosis would be Imbalanced nutrition: less than body requirements.

The perinatal nurse is giving discharge instructions to a woman after suction curettage secondary to a hydatidiform mole. The woman asks why she must take oral contraceptives for the next 12 months. The best response from the nurse would be: a.If you get pregnant within 1 year, the chance of a successful pregnancy is very small. Therefore, if you desire a future pregnancy, it would be better for you to use the most reliable method of contraception available. b.The major risk to you after a molar pregnancy is a type of cancer that can be diagnosed only by measuring the same hormone that your body produces during pregnancy. If you were to get pregnant, it would make the diagnosis of this cancer more difficult. c.If you can avoid a pregnancy for the next year, the chance of developing a second molar pregnancy is rare. Therefore, to improve your chance of a successful pregnancy, it is better not to get pregnant at this time. d.Oral contraceptives are the only form of birth control that will prevent a recurrence of a molar pregnancy.

ANS: B This is an accurate statement. b-Human chorionic gonadotropin (hCG) levels will be drawn for 1 year to ensure that the mole is completely gone. There is an increased chance of developing choriocarcinoma after the development of a hydatidiform mole. The goal is to achieve a zero hCG level. If the woman were to become pregnant, it could obscure the presence of the potentially carcinogenic cells. Women should be instructed to use birth control for 1 year after treatment for a hydatidiform mole. The rationale for avoiding pregnancy for 1 year is to ensure that carcinogenic cells are not present. Any contraceptive method except an intrauterine device is acceptable.

Nurses, certified nurse-midwives, and other advanced practice nurses have the knowledge and expertise to assist women in making informed choices regarding contraception. A multidisciplinary approach should ensure that the womans social, cultural, and interpersonal needs are met. Which action should the nurse take first when meeting with a new client to discuss contraception? a.Obtain data about the frequency of coitus. b.Determine the womans level of knowledge about contraception and commitment to any particular method. c.Assess the womans willingness to touch her genitals and cervical mucus. d.Evaluate the womans contraceptive life plan.

ANS: B This is the primary step of this nursing assessment and necessary before completing the process and moving on to a nursing diagnosis. Once the clients level of knowledge is determined, the nurse can interact with the woman to compare options, reliability, cost, comfort level, protection from sexually transmitted infections, and a partners willingness to participate. Although important, obtaining data about the frequency of coitus is not the first action that the nurse should undertake when completing an assessment. Data should include not only the frequency of coitus but also the number of sexual partners, level of contraceptive involvement, and partners objections. Assessing the womans willingness to touch herself is a key factor for the nurse to discuss should the client express interest in using one of the fertility awareness methods of contraception. The nurse must be aware of the clients plan regarding whether she is attempting to prevent conception, delay conception, or conceive.

27. A laboring woman with no known risk factors suddenly experiences spontaneous rupture of membranes (ROM). The fluid consists of bright red blood. Her contractions are consistent with her current stage of labor. There is no change in uterine resting tone. The fetal heart rate begins to decline rapidly after the ROM. The nurse should suspect the possibility of: a.Placenta previa. b.Vasa previa. c.Severe abruptio placentae. d.Disseminated intravascular coagulation (DIC).

ANS: B Vasa previa is the result of a velamentous insertion of the umbilical cord. The umbilical vessels are not surrounded by Wharton jelly and have no supportive tissue. They are at risk for laceration at any time, but laceration occurs most frequently during ROM. The sudden appearance of bright red blood at the time of ROM and a sudden change in the fetal heart rate without other known risk factors should immediately alert the nurse to the possibility of vasa previa. The presence of placenta previa most likely would be ascertained before labor and would be considered a risk factor for this pregnancy. In addition, if the woman had a placenta previa, it is unlikely that she would be allowed to pursue labor and a vaginal birth. With the presence of severe abruptio placentae, the uterine tonicity would typically be tetanus (i.e., a boardlike uterus). DIC is a pathologic form of diffuse clotting that consumes large amounts of clotting factors and causes widespread external bleeding, internal bleeding, or both. DIC is always a secondary diagnosis, often associated with obstetric risk factors such as HELLP syndrome. This woman did not have any prior risk factors.

Which nursing intervention is necessary before a second-trimester transabdominal ultrasound? a.Place the woman NPO for 12 hours. b.Instruct the woman to drink 1 to 2 quarts of water. c.Administer an enema. d.Perform an abdominal preparation.

ANS: B When the uterus is still in the pelvis, visualization may be difficult. It is necessary to perform the test when the woman has a full bladder, which provides a window through which the uterus and its contents can be viewed. The woman needs a full bladder to elevate the uterus; therefore being NPO is not appropriate. Neither an enema nor an abdominal preparation is necessary for this procedure.

Women with hyperemesis gravidarum: a.Are a majority, because 80% of all pregnant women suffer from it at some time. b.Have vomiting severe and persistent enough to cause weight loss, dehydration, and electrolyte imbalance. c.Need intravenous (IV) fluid and nutrition for most of their pregnancy. d.Often inspire similar, milder symptoms in their male partners and mothers.

ANS: B Women with hyperemesis gravidarum have severe vomiting; however, treatment for several days sets things right in most cases. Although 80% of pregnant women experience nausea and vomiting, fewer than 1% (0.5%) proceed to this severe level. IV administration may be used at first to restore fluid levels, but it is seldom needed for very long. Women suffering from this condition want sympathy because some authorities believe that difficult relationships with mothers and/or partners may be the cause.

Maternal hypotension is a potential side effect of regional anesthesia and analgesia. What nursing interventions could you use to raise the clients blood pressure (Select all that apply)? a. Place the woman in a supine position. b. Place the woman in a lateral position. c. Increase intravenous (IV) fluids. d. Administer oxygen. e. Perform a vaginal examination.

ANS: B, C, D Nursing interventions for maternal hypotension arising from analgesia or anesthesia include turning the woman to a lateral position, increasing IV fluids, administering oxygen via face mask, elevating the womans legs, notifying the physician, administering an IV vasopressor, and monitoring the maternal and fetal status at least every 5 minutes until these are stable. Placing the client in a supine position would cause venous compression, thereby limiting blood flow to and oxygenation of the placenta and fetus. A sterile vaginal examination has no bearing on maternal blood pressure.

Autoimmune disorders often occur during pregnancy because a large percentage of women with an autoimmune disorder are of childbearing age. Identify all disorders that fall into the category of collagen vascular disease. a.Multiple sclerosis b.Systemic lupus erythematosus c.Antiphospholipid syndrome d.Rheumatoid arthritis e.Myasthenia gravis

ANS: B, C, D, E Multiple sclerosis is not an autoimmune disorder. This patchy demyelinization of the spinal cord may be a viral disorder. Autoimmune disorders (collagen vascular disease) make up a large group of conditions that disrupt the function of the immune system of the body. They include those listed, as well as systemic sclerosis.

The nurse providing care for a woman with gestational diabetes understands that a laboratory test for glycosylated hemoglobin Alc: a.Is now done for all pregnant women, not just those with or likely to have diabetes. b.Is a snapshot of glucose control at the moment. c.Would be considered evidence of good diabetes control with a result of 5% to 6%. d.Is done on the patients urine, not her blood.

ANS: C A score of 5% to 6% indicates good control. This is an extra test for diabetic women, not one done for all pregnant women. This test defines glycemic control over the previous 4 to 6 weeks. Glycosylated hemoglobin level tests are done on the blood.

25. Methotrexate is recommended as part of the treatment plan for which obstetric complication? a.Complete hydatidiform mole b.Missed abortion c.Unruptured ectopic pregnancy d.Abruptio placentae

ANS: C Methotrexate is an effective, nonsurgical treatment option for a hemodynamically stable woman whose ectopic pregnancy is unruptured and less than 4 cm in diameter. Methotrexate is not indicated or recommended as a treatment option for complete hydatidiform mole, missed abortion, and abruptio placentae.

A woman arrives for evaluation of her symptoms, which include a missed period, adnexal fullness, tenderness, and dark red vaginal bleeding. On examination the nurse notices an ecchymotic blueness around the womans umbilicus and recognizes this assessment finding as: a.Normal integumentary changes associated with pregnancy. b.Turners sign associated with appendicitis. c.Cullens sign associated with a ruptured ectopic pregnancy. d.Chadwicks sign associated with early pregnancy.

ANS: C Cullens sign, the blue ecchymosis seen in the umbilical area, indicates hematoperitoneum associated with an undiagnosed ruptured intraabdominal ectopic pregnancy. Linea nigra on the abdomen is the normal integumentary change associated with pregnancy. It manifests as a brown, pigmented, vertical line on the lower abdomen. Turners sign is ecchymosis in the flank area, often associated with pancreatitis. Chadwicks sign is the blue-purple color of the cervix that may be seen during or around the eighth week of pregnancy.

With regard to amniocentesis, nurses should be aware that: a.Because of new imaging techniques, amniocentesis is now possible in the first trimester. b.Despite the use of ultrasound, complications still occur in the mother or infant in 5% to 10% of cases. c.The shake test, or bubble stability test, is a quick means of determining fetal maturity. d.The presence of meconium in the amniotic fluid is always cause for concern.

ANS: C Diluted fluid is mixed with ethanol and shaken. After 15 minutes, the bubbles tell the story. Amniocentesis is possible after the fourteenth week of pregnancy when the uterus becomes an abdominal organ. Complications occur in less than 1% of cases; many have been minimized or eliminated through the use of ultrasound. Meconium in the amniotic fluid before the beginning of labor is not usually a problem.

Which nursing assessment indicates that a woman who is in second-stage labor is almost ready to give birth? a.The fetal head is felt at 0 station during vaginal examination. b.Bloody mucus discharge increases. c.The vulva bulges and encircles the fetal head. d.The membranes rupture during a contraction

ANS: C During the active pushing (descent) phase, the woman has strong urges to bear down as the presenting part of the fetus descends and presses on the stretch receptors of the pelvic floor. The vulva stretches and begins to bulge encircling the fetal head. Birth of the head occurs when the station is +4. A 0 station indicates engagement. Bloody show occurs throughout the labor process and is not an indication of an imminent birth. Rupture of membranes can occur at any time during the labor process and does not indicate an imminent birth.

While providing care to a patient in active labor, the nurse should instruct the woman that: a.The supine position commonly used in the United States increases blood flow. b.The all fours position, on her hands and knees, is hard on her back. c.Frequent changes in position will help relieve her fatigue and increase her comfort. d.In a sitting or squatting position, her abdominal muscles will have to work harder.

ANS: C Frequent position changes relieve fatigue, increase comfort, and improve circulation. Blood flow can be compromised in the supine position; any upright position benefits cardiac output. The all fours position is used to relieve backache in certain situations. In a sitting or squatting position, the abdominal muscles work in greater harmony with uterine contractions

Nurses with an understanding of cultural differences regarding likely reactions to pain may be better able to help clients. Nurses should know that _____ women may be stoic until late in labor, when they may become vocal and request pain relief. a.Chinese b.Arab or Middle Eastern c.Hispanic d.African-American

ANS: C Hispanic women may be stoic early and more vocal and ready for medications later. Chinese women may not show reactions to pain. Medical interventions must be offered more than once. Arab or Middle Eastern women may be vocal in response to labor pain from the start. They may prefer pain medications. African-American women may express pain openly; use of medications for pain is more likely to vary with the individual.

The laboring woman who imagines her body opening to let the baby out is using a mental technique called: a.Dissociation. b.Effleurage. c.Imagery. d.Distraction.

ANS: C Imagery is a technique of visualizing images that will assist the woman in coping with labor. Dissociation helps the woman learn to relax all muscles except those that are working. Effleurage is self-massage. Distraction can be used in the early latent phase by having the woman engage in another activity.

In providing nutritional counseling for the pregnant woman experiencing cholecystitis, the nurse would: a.Assess the womans dietary history for adequate calories and proteins. b.Instruct the woman that the bulk of calories should come from proteins. c.Instruct the woman to eat a low-fat diet and avoid fried foods. d.Instruct the woman to eat a low-cholesterol, low-salt diet.

ANS: C Instructing the woman to eat a low-fat diet and avoid fried foods is appropriate nutritional counseling for this client. Caloric and protein intake do not predispose a woman to the development of cholecystitis. The woman should be instructed to limit protein intake and choose foods that are high in carbohydrates. A low-cholesterol diet may be the result of limiting fats. However, a low-salt diet is not indicated.

Nursing intervention for the pregnant diabetic patient is based on the knowledge that the need for insulin: a.Increases throughout pregnancy and the postpartum period. b.Decreases throughout pregnancy and the postpartum period. c.Varies depending on the stage of gestation. d.Should not change because the fetus produces its own insulin.

ANS: C Insulin needs decrease during the first trimester, when nausea, vomiting, and anorexia are a factor. They increase during the second and third trimesters, when the hormones of pregnancy create insulin resistance in maternal cells. Insulin needs increase during the second and third trimesters, when the hormones of pregnancy create insulin resistance in maternal cells. The insulin needs change throughout the different stages of pregnancy.

Which minerals and vitamins usually are recommended to supplement a pregnant womans diet? a.Fat-soluble vitamins A and D b.Water-soluble vitamins C and B6 c.Iron and folate d.Calcium and zinc

ANS: C Iron generally should be supplemented, and folic acid supplements often are needed because folate is so important. Fat-soluble vitamins should be supplemented as a medical prescription, as vitamin D might be for lactose-intolerant women. Water-soluble vitamin C sometimes is consumed in excess naturally; vitamin B6 is prescribed only if the woman has a very poor diet. Zinc sometimes is supplemented. Most women obtain enough calcium through their regular diet.

Metabolic changes throughout pregnancy that affect glucose and insulin in the mother and the fetus are complicated but important to understand. Nurses should understand that: a.Insulin crosses the placenta to the fetus only in the first trimester, after which the fetus secretes its own. b.Women with insulin-dependent diabetes are prone to hyperglycemia during the first trimester because they are consuming more sugar. c.During the second and third trimesters, pregnancy exerts a diabetogenic effect that ensures an abundant supply of glucose for the fetus. d.Maternal insulin requirements steadily decline during pregnancy.

ANS: C Pregnant women develop increased insulin resistance during the second and third trimesters. Insulin never crosses the placenta; the fetus starts making its own insulin around the tenth week. As a result of normal metabolic changes during pregnancy, insulin-dependent women are prone to hypoglycemia (low levels). Maternal insulin requirements may double or quadruple by the end of pregnancy.

Which factor is known to increase the risk of gestational diabetes mellitus? a.Underweight before pregnancy b.Maternal age younger than 25 years c.Previous birth of large infant d.Previous diagnosis of type 2 diabetes mellitus

ANS: C Previous birth of a large infant suggests gestational diabetes mellitus. Obesity (BMI of 30 or greater) creates a higher risk for gestational diabetes. A woman younger than 25 years generally is not at risk for gestational diabetes mellitus. The person with type 2 diabetes mellitus already has diabetes and will continue to have it after pregnancy. Insulin may be required during pregnancy because oral hypoglycemia drugs are contraindicated during pregnancy.

A woman with asthma is experiencing a postpartum hemorrhage. Which drug would not be used to treat her bleeding because it may exacerbate her asthma? a.Pitocin b.Nonsteroidal antiinflammatory drugs (NSAIDs) c.Hemabate d.Fentanyl

ANS: C Prostaglandin derivatives should not be used to treat women with asthma, because they may exacerbate symptoms. Pitocin would be the drug of choice to treat this womans bleeding because it would not exacerbate her asthma. NSAIDs are not used to treat bleeding. Fentanyl is used to treat pain, not bleeding.

A primigravida is being monitored in her prenatal clinic for preeclampsia. What finding should concern her nurse? a.Blood pressure (BP) increase to 138/86 mm Hg b.Weight gain of 0.5 kg during the past 2 weeks c.A dipstick value of 3+ for protein in her urine d.Pitting pedal edema at the end of the day

ANS: C Proteinuria is defined as a concentration of 1+ or greater via dipstick measurement. A dipstick value of 3+ should alert the nurse that additional testing or assessment should be made. Generally, hypertension is defined as a BP of 140/90 or an increase in systolic pressure of 30 mm Hg or in diastolic pressure of 15 mm Hg. Preeclampsia may be manifested as a rapid weight gain of more than 2 kg in 1 week. Edema occurs in many normal pregnancies and in women with preeclampsia. Therefore, the presence of edema is no longer considered diagnostic of preeclampsia.

_____ use/abuse during pregnancy causes vasoconstriction and decreased placental perfusion, resulting in maternal and neonatal complications. a.Alcohol b.Caffeine c.Tobacco d.Chocolate

ANS: C Smoking in pregnancy is known to cause a decrease in placental perfusion and has serious health risks, including bleeding complications, low birth weight, prematurity, miscarriage, stillbirth, and sudden infant death syndrome. Prenatal alcohol exposure is the single greatest preventable cause of mental retardation. Alcohol use during pregnancy can cause high blood pressure, miscarriage, premature birth, stillbirth, and anemia. Caffeine and chocolate may safely be consumed in small quantities during pregnancy.

Which condition would not be classified as a bleeding disorder in late pregnancy? a.Placenta previa. b.Abruptio placentae. c.Spontaneous abortion. d.Cord insertion.

ANS: C Spontaneous abortion is another name for miscarriage; by definition it occurs early in pregnancy. Placenta previa is a cause of bleeding disorders in later pregnancy. Abruptio placentae is a cause of bleeding disorders in later pregnancy. Cord insertion is a cause of bleeding disorders in later pregnancy.

Nurses should be aware that HELLP syndrome: a.Is a mild form of preeclampsia. b.Can be diagnosed by a nurse alert to its symptoms. c.Is characterized by hemolysis, elevated liver enzymes, and low platelets. d.Is associated with preterm labor but not perinatal mortality.

ANS: C The acronym HELLP stands for hemolysis (H), elevated liver enzymes (EL), and low platelets (LP). HELLP syndrome is a variant of severe preeclampsia. HELLP syndrome is difficult to identify because the symptoms often are not obvious. It must be diagnosed in the laboratory. Preterm labor is greatly increased, and so is perinatal mortality.

A pregnant woman has been receiving a magnesium sulfate infusion for treatment of severe preeclampsia for 24 hours. On assessment the nurse finds the following vital signs: temperature of 37.3 C, pulse rate of 88 beats/min, respiratory rate of 10 breaths/min, blood pressure (BP) of 148/90 mm Hg, absent deep tendon reflexes, and no ankle clonus. The client complains, Im so thirsty and warm. The nurse: a.Calls for a stat magnesium sulfate level. b.Administers oxygen. c.Discontinues the magnesium sulfate infusion. d.Prepares to administer hydralazine.

ANS: C The client is displaying clinical signs and symptoms of magnesium toxicity. Magnesium should be discontinued immediately. In addition, calcium gluconate, the antidote for magnesium, may be administered. Hydralazine is an antihypertensive commonly used to treat hypertension in severe preeclampsia. Typically it is administered for a systolic BP greater than 160 mm Hg or a diastolic BP greater than 110 mm Hg.

While taking a diet history, the nurse might be told that the expectant mother has cravings for ice chips, cornstarch, and baking soda. This represents a nutritional problem known as: a.Preeclampsia. b.Pyrosis. c.Pica. d.Purging.

ANS: C The consumption of foods low in nutritional value or of nonfood substances (e.g., dirt, laundry starch) is called pica.

The most common neurologic disorder accompanying pregnancy is: a.Eclampsia. b.Bells palsy. c.Epilepsy. d.Multiple sclerosis.

ANS: C The effects of pregnancy on epilepsy are unpredictable. Eclampsia sometimes may be confused with epilepsy, which is the most common neurologic disorder accompanying pregnancy. Bells palsy is a form of facial paralysis. Multiple sclerosis is a patchy demyelinization of the spinal cord that does not affect the normal course of pregnancy or birth.

When assessing the fetus using Leopold maneuvers, the nurse feels a round, firm, movable fetal part in the fundal portion of the uterus and a long, smooth surface in the mothers right side close to midline. What is the likely position of the fetus? a.ROA b.LSP c.RSA d.LOA

ANS: C The fetus is positioned anteriorly in the right side of the maternal pelvis with the sacrum as the presenting part. RSA is the correct three-letter abbreviation to indicate this fetal position. The first letter indicates the presenting part in either the right or left side of the maternal pelvis. The second letter indicates the anatomic presenting part of the fetus. The third letter stands for the location of the presenting part in relation to the anterior, posterior, or transverse portion of the maternal pelvis. Palpation of a round, firm fetal part in the fundal portion of the uterus would be the fetal head, indicating that the fetus is in a breech position with the sacrum as the presenting part in the maternal pelvis. Palpation of the fetal spine along the mothers right side denotes the location of the presenting part in the mothers pelvis. The ability to palpate the fetal spine indicates that the fetus is anteriorly positioned in the maternal pelvis.

A woman in labor has just received an epidural block. The most important nursing intervention is to: a.Limit parenteral fluids. b.Monitor the fetus for possible tachycardia. c.Monitor the maternal blood pressure for possible hypotension. d.Monitor the maternal pulse for possible bradycardia.

ANS: C The most important nursing intervention for a woman who has received an epidural block is to monitor the maternal blood pressure frequently for signs of hypotension. Intravenous fluids are increased for a woman receiving an epidural, to prevent hypotension. The nurse observes for signs of fetal bradycardia. The nurse monitors for signs of maternal tachycardia secondary to hypotension.

With regard to factors that affect how the fetus moves through the birth canal, nurses should be aware that: a.The fetal attitude describes the angle at which the fetus exits the uterus. b.Of the two primary fetal lies, the horizontal lie is that in which the long axis of the fetus is parallel to the long axis of the mother. c.The normal attitude of the fetus is called general flexion. d.The transverse lie is preferred for vaginal birth.

ANS: C The normal attitude of the fetus is general flexion. The fetal attitude is the relation of fetal body parts to one another. The horizontal lie is perpendicular to the mother; in the longitudinal (or vertical) lie the long axes of the fetus and the mother are parallel. Vaginal birth cannot occur if the fetus stays in a transverse lie.

A woman has requested an epidural for her pain. She is 5 cm dilated and 100% effaced. The baby is in a vertex position and is engaged. The nurse increases the womans intravenous fluid for a preprocedural bolus. She reviews her laboratory values and notes that the womans hemoglobin is 12 g/dL, hematocrit is 38%, platelets are 67,000, and white blood cells (WBCs) are 12,000/mm3. Which factor would contraindicate an epidural for the woman? a.She is too far dilated. b.She is anemic. c.She has thrombocytopenia. d.She is septic.

ANS: C The platelet count indicates a coagulopathy, specifically, thrombocytopenia (low platelets), which is a contraindication to epidural analgesia/anesthesia. Typically epidural analgesia/anesthesia is used in the laboring woman when a regular labor pattern has been achieved, as evidenced by progressive cervical change. The laboratory values show that the womans hemoglobin and hematocrit are in the normal range and show a slight increase in the WBC count that is not uncommon in laboring women.

When providing care to the prenatal patient, the nurse understands that pica is defined as: a.Intolerance of milk products b.Iron deficiency anemia c.Ingestion of nonfood substances d.Episodes of anorexia and vomiting

ANS: C The practice of eating substances not normally thought of as food is called pica. Clay or dirt and solid laundry starch are the substances most commonly ingested. Intolerance of milk products is referred to as lactose intolerance. Pica may produce iron deficiency anemia if proper nutrition is decreased. Pica is not related to anorexia and vomiting.

If a patients normal prepregnancy diet contains 45 g of protein daily, how many more grams of protein should she consume per day during pregnancy? a.5 b.10 c.25 d.30

ANS: C The recommended intake of protein for the pregnant woman is 70 g. Intakes of 5, 10, or 15 g would be inadequate to meet protein needs during pregnancy. A protein intake of 30 g is more than is necessary and would add extra calories.

With regard to the turns and other adjustments of the fetus during the birth process, known as the mechanism of labor, nurses should be aware that: a.The seven critical movements must progress in a more or less orderly sequence. b.Asynclitism sometimes is achieved by means of the Leopold maneuver. c.The effects of the forces determining descent are modified by the shape of the womans pelvis and the size of the fetal head. d.At birth the baby is said to achieve restitution (i.e., a return to the C-shape of the womb).

ANS: C The size of the maternal pelvis and the ability of the fetal head to mold also affect the process. The seven identifiable movements of the mechanism of labor occur in combinations simultaneously, not in precise sequences.Asynclitism is the deflection of the babys head; the Leopold maneuver is a means of judging descent by palpating the mothers abdomen. Restitution is the rotation of the babys head after the infant is born.

A couple comes in for an infertility workup, having attempted to get pregnant for 2 years. The woman, 37, has always had irregular menstrual cycles but is otherwise healthy. The man has fathered two children from a previous marriage and had a vasectomy reversal 2 years ago. The man has had two normal semen analyses, but the sperm seem to be clumped together. What additional test is needed? a.Testicular biopsy b.Antisperm antibodies c.Follicle-stimulating hormone (FSH) level d.Examination for testicular infection

ANS: C The woman has irregular menstrual cycles. The scenario does not indicate that she has had any testing related to this irregularity. Hormone analysis is performed to assess endocrine function of the hypothalamic-pituitary-ovarian axis when menstrual cycles are absent or irregular. Determination of blood levels of prolactin, FSH, luteinizing hormone (LH), estradiol, progesterone, and thyroid hormones may be necessary to diagnose the cause of irregular menstrual cycles. A testicular biopsy would be indicated only in cases of azoospermia (no sperm cells) or severe oligospermia (low number of sperm cells). Antisperm antibodies are produced by a man against his own sperm. This is unlikely to be the case here because the man has already produced children. Examination for testicular infection would be done before semen analysis. Infection would affect spermatogenesis.

In the past, factors to determine whether a woman was likely to develop a high risk pregnancy were evaluated primarily from a medical point of view. A broader, more comprehensive approach to high-risk pregnancy has been adopted today. There are now four categories based on threats to the health of the woman and the outcome of pregnancy. Which of the following is not one of these categories? a.Biophysical b.Psychosocial c.Geographic d.Environmental

ANS: C This category is correctly referred to as sociodemographic risk. These factors stem from the mother and her family. Ethnicity may be one of the risks to pregnancy; however, it is not the only factor in this category. Low income, lack of prenatal care, age, parity, and marital status also are included. Biophysical is one of the broad categories used for determining risk. These include genetic considerations, nutritional status, and medical and obstetric disorders. Psychosocial risks include smoking, caffeine, drugs, alcohol, and psychologic status. All of these adverse lifestyles can have a negative effect on the health of the mother or fetus. Environmental risks are risks that can affect both fertility and fetal development. These include infections, chemicals, radiation, pesticides, illicit drugs, and industrial pollutants.

A pregnant womans diet consists almost entirely of whole grain breads and cereals, fruits, and vegetables. The nurse would be most concerned about this womans intake of: a.Calcium. b.Protein. c.Vitamin B12. d.Folic acid.

ANS: C This diet is consistent with that followed by a strict vegetarian (vegan). Vegans consume only plant products. Because vitamin B12 is found in foods of animal origin, this diet is deficient in vitamin B12.

A woman in labor is breathing into a mouthpiece just before the start of her regular contractions. As she inhales, a valve opens, and gas is released. She continues to inhale the gas slowly and deeply until the contraction starts to subside. When the inhalation stops, the valve closes. This procedure is: a.Not used much anymore. b.Likely to be used in the second stage of labor but not in the first stage. c.An application of nitrous oxide. d.A prelude to cesarean birth.

ANS: C This is an application of nitrous oxide, which could be used in either the first or second stage of labor (or both) as part of the preparation for a vaginal birth. Nitrous oxide is self-administered and found to be very helpful.

A 27-year-old pregnant woman had a preconceptual body mass index (BMI) of 18.0. The nurse knows that this womans total recommended weight gain during pregnancy should be at least: a.20 kg (44 lb). b.16 kg (35 lb). c.12.5 kg (27.5 lb). d.10 kg (22 lb).

ANS: C This woman has a normal BMI and should gain 11.5 to 16 kg during pregnancy. A weight gain of 20 kg would be unhealthy for most women. A weight gain 35 lb is the high end of the range of weight this woman should gain in her pregnancy. A weight gain of 22 lb would be appropriate for an obese woman.

Preeclampsia is a unique disease process related only to human pregnancy. The exact cause of this condition continues to elude researchers. The American College of Obstetricians and Gynecologists has developed a comprehensive list of risk factors associated with the development of preeclampsia. Which client exhibits the greatest number of these risk factors? a.A 30-year-old obese Caucasian with her third pregnancy b.A 41-year-old Caucasian primigravida c.An African-American client who is 19 years old and pregnant with twins d.A 25-year-old Asian-American whose pregnancy is the result of donor insemination

ANS: C Three risk factors are present for this woman. She is of African-American ethnicity, is at the young end of the age distribution, and has a multiple pregnancy. In planning care for this client the nurse must monitor blood pressure frequently and teach the woman regarding early warning signs. The 30-year-old client only has one known risk factor, obesity. Age distribution appears to be U-shaped, with women less than 20 years and more than 40 years being at greatest risk. Preeclampsia continues to be seen more frequently in primigravidas; this client is a multigravida woman. Two risk factors are present for the 41-year-old client. Her age and status as a primigravida put her at increased risk for preeclampsia. Caucasian women are at a lower risk than African-American women. The Asian-American client exhibits only one risk factor. Pregnancies that result from donor insemination, oocyte donation, and embryo donation are at an increased risk of developing preeclampsia.

A woman will be taking oral contraceptives using a 28-day pack. The nurse should advise this woman to protect against pregnancy by: a.Limiting sexual contact for one cycle after starting the pill. b.Using condoms and foam instead of the pill for as long as she takes an antibiotic. c.Taking one pill at the same time every day. d.Throwing away the pack and using a backup method if she misses two pills during week 1 of her cycle.

ANS: C To maintain adequate hormone levels for contraception and to enhance compliance, clients should take oral contraceptives at the same time each day. If contraceptives are to be started at any time other than during normal menses or within 3 weeks after birth or abortion, another method of contraception should be used through the first week to prevent the risk of pregnancy. Taken exactly as directed, oral contraceptives prevent ovulation, and pregnancy cannot occur. No strong pharmacokinetic evidence indicates a link between the use of broad-spectrum antibiotics and altered hormone levels in oral contraceptive users. If the client misses two pills during week 1, she should take two pills a day for 2 days, finish the package, and use a backup method the next 7 consecutive days.

A woman had unprotected intercourse 36 hours ago and is concerned that she may become pregnant because it is her fertile time. She asks the nurse about emergency contraception. The nurse tells her that: a.It is too late; she needed to begin treatment within 24 hours after intercourse. b.Preven, an emergency contraceptive method, is 98% effective at preventing pregnancy. c.An over-the-counter antiemetic can be taken 1 hour before each contraceptive dose to prevent nausea and vomiting. d.The most effective approach is to use a progestin-only preparation.

ANS: C To minimize the side effect of nausea that occurs with high doses of estrogen and progestin, the woman can take an over-the-counter antiemetic 1 hour before each dose. Emergency contraception is used within 72 hours of unprotected intercourse to prevent pregnancy. Postcoital contraceptive use is 74% to 90% effective at preventing pregnancy. Oral emergency contraceptive regimens may include progestin-only and estrogen-progestin pills. Women with contraindications to estrogen use should use progestin-only pills.

A couple is trying to cope with an infertility problem. They want to know what they can do to preserve their emotional equilibrium. The nurses most appropriate response is: a.Tell your friends and family so they can help you. b.Talk only to other friends who are infertile because only they can help. c.Get involved with a support group. Ill give you some names. d.Start adoption proceedings immediately because it is very difficult to obtain an infant.

ANS: C Venting negative feelings may unburden the couple. A support group may provide a safe haven for the couple to share their experiences and gain insight from others experiences. Although talking about their feelings may unburden them of negative feelings, infertility can be a major stressor that affects the couples relationships with family and friends. Limiting their interactions to other infertile couples may be a beginning point for addressing psychosocial needs, but depending on where the other couple is in their own recovery process, this may or may not help them. The statement about adoption proceedings is not supportive of the psychosocial needs of this couple and may be detrimental to their well-being.

In caring for a pregnant woman with sickle cell anemia, the nurse is aware that signs and symptoms of sickle cell crisis include: a.Anemia. b.Endometritis. c.Fever and pain. d.Urinary tract infection.

ANS: C Women with sickle cell anemia have recurrent attacks (crisis) of fever and pain, most often in the abdomen, joints, and extremities. These attacks are attributed to vascular occlusion when RBCs assume the characteristic sickled shape. Crises are usually triggered by dehydration, hypoxia, or acidosis. Women with sickle cell anemia are not iron deficient. Therefore, routine iron supplementation, even that found in prenatal vitamins, should be avoided in order to prevent iron overload. Women with sickle cell trait usually are at greater risk for postpartum endometritis (uterine wall infection); however, this is not likely to occur in pregnancy and is not a sign of crisis. These women are at an increased risk for UTIs; however, this is not an indication of sickle cell crisis.

A woman with gestational diabetes has had little or no experience reading and interpreting glucose levels. She shows the nurse her readings for the past few days. Which one should the nurse tell her indicates a need for adjustment (insulin or sugar)? a.75 mg/dL before lunch. This is low; better eat now. b.115 mg/dL 1 hour after lunch. This is a little high; maybe eat a little less next time. c.115 mg/dL 2 hours after lunch; This is too high; it is time for insulin. d.60 mg/dL just after waking up from a nap. This is too low; maybe eat a snack before going to sleep.

ANS: D 60 mg/dL after waking from a nap is too low. During hours of sleep glucose levels should not be less than 70 mg/dL. Snacks before sleeping can be helpful. The premeal acceptable range is 65 to 95 mg/dL. The readings 1 hour after a meal should be less than 140 mg/dL. Two hours after eating, the readings should be less than 120 mg/dL.

A woman has come to the clinic for preconception counseling because she wants to start trying to get pregnant in 3 months. She can expect the following advice: a.Discontinue all contraception now. b.Lose weight so that you can gain more during pregnancy. c.You may take any medications you have been taking regularly. d.Make sure that you include adequate folic acid in your diet.

ANS: D A healthy diet before conception is the best way to ensure that adequate nutrients are available for the developing fetus. A womans folate or folic acid intake is of particular concern in the periconception period. Neural tube defects are more common in infants of women with a poor folic acid intake. Depending on the type of contraception used, discontinuing all contraception may not be appropriate advice. Losing weight is not appropriate advice. Depending on the type of medication the woman is taking, continuing its use may not be appropriate.

Which basic type of pelvis includes the correct description and percentage of occurrence in women? a.Gynecoid: classic female; heart shaped; 75% b.Android: resembling the male; wider oval; 15% c.Anthropoid: resembling the ape; narrower; 10% d.Platypelloid: flattened, wide, shallow; 3%

ANS: D A platypelloid pelvis is flattened, wide, and shallow; about 3% of women have this shape. The gynecoid shape is the classical female shape, slightly ovoid and rounded; about 50% of women have this shape. An android, or malelike, pelvis is heart shaped; about 23% of women have this shape. An anthropoid, or apelike, pelvis is oval and wider; about 24% of women have this shape.

Which pregnant woman should restrict her weight gain during pregnancy? a.Woman pregnant with twins b.Woman in early adolescence c.Woman shorter than 62 inches or 157 cm d.Woman who was 20 pounds overweight before pregnancy

ANS: D A weight gain of 5 to 9 kg will provide sufficient nutrients for the fetus. Overweight and obese women should be advised to lose weight before conception to achieve the best pregnancy outcomes. A higher weight gain in twin gestations may help prevent low birth weights. Adolescents need to gain weight toward the higher acceptable range, which provides for their own growth as well as for fetal growth. In the past, women of short stature were advised to restrict their weight gain; however, evidence to support these guidelines has not been found.

Although remarkable developments have occurred in reproductive medicine, assisted reproductive therapies are associated with numerous legal and ethical issues. Nurses can provide accurate information about the risks and benefits of treatment alternatives so couples can make informed decisions about their choice of treatment. Which issue would not need to be addressed by an infertile couple before treatment? a.Risks of multiple gestation b.Whether or how to disclose the facts of conception to offspring c.Freezing embryos for later use d.Financial ability to cover the cost of treatment

ANS: D Although the method of payment is important, obtaining this information is not the responsibility of the nurse. Many states have mandated some form of insurance to assist couples with coverage for infertility. Risk of multiple gestation is a risk of treatment of which the couple needs to be aware. To minimize the chance of multiple gestation, generally only three or fewer embryos are transferred. The couple should be informed that there may be a need for multifetal reduction. Nurses can provide anticipatory guidance on this matter. Depending on the therapy chosen, there may be a need for donor oocytes, sperm, embryos, or a surrogate mother. Couples who have excess embryos frozen for later transfer must be fully informed before consenting to the procedure. A decision must be made regarding the disposal of embryos in the event of death or divorce or if the couple no longer wants the embryos at a future time.

An 18-year-old pregnant woman, gravida 1, is admitted to the labor and birth unit with moderate contractions every 5 minutes that last 40 seconds. The woman states, My contractions are so strong that I dont know what to do with myself. The nurse should: a.Assess for fetal well-being. b.Encourage the woman to lie on her side. c.Disturb the woman as little as possible. d.Recognize that pain is personalized for each individual.

ANS: D Each womans pain during childbirth is unique and is influenced by a variety of physiologic, psychosocial, and environmental factors. A critical issue for the nurse is how support can make a difference in the pain of the woman during labor and birth. Assessing for fetal well-being includes no information that would indicate fetal distress or a logical reason to be overly concerned about the well-being of the fetus. The left lateral position is used to alleviate fetal distress, not maternal stress. The nurse has an obligation to provide physical, emotional, and psychosocial care and support to the laboring woman. This client clearly needs support.

A woman in week 34 of pregnancy reports that she is very uncomfortable because of heartburn. The nurse would suggest that the woman: a.Substitute other calcium sources for milk in her diet. b.Lie down after each meal. c.Reduce the amount of fiber she consumes. d.Eat five small meals daily.

ANS: D Eating small, frequent meals may help with heartburn, nausea, and vomiting. Substituting other calcium sources for milk, lying down after eating, and reducing fiber intake are inappropriate dietary suggestions for all pregnant women and do not alleviate heartburn.

In order to care for obstetric patients adequately, the nurse understands that labor contractions facilitate cervical dilation by: a.Contracting the lower uterine segment. b.Enlarging the internal size of the uterus. c.Promoting blood flow to the cervix. d.Pulling the cervix over the fetus and amniotic sac.

ANS: D Effective uterine contractions pull the cervix upward at the same time that the fetus and amniotic sac are pushed downward. The contractions are stronger at the fundus. The internal size becomes smaller with the contractions; this helps to push the fetus down. Blood flow decreases to the uterus during a contraction.

In the current practice of childbirth preparation, emphasis is placed on: a.The Dick-Read (natural) childbirth method. b.The Lamaze (psychoprophylactic) method. c.The Bradley (husband-coached) method. d.Having expectant parents attend childbirth preparation in any or no specific method.

ANS: D Encouraging expectant parents to attend childbirth preparation class is most important because preparation increases a womans confidence and thus her ability to cope with labor and birth. Although still popular, the method format of classes is being replaced with other offerings such as Hypnobirthing and Birthing from Within.

A woman arrives at the clinic seeking confirmation that she is pregnant. The following information is obtained: She is 24 years old with a body mass index (BMI) of 17.5. She admits to having used cocaine several times during the past year and drinks alcohol occasionally. Her blood pressure (BP) is 108/70 mm Hg, her pulse rate is 72 beats/min, and her respiratory rate is 16 breaths/min. The family history is positive for diabetes mellitus and cancer. Her sister recently gave birth to an infant with a neural tube defect (NTD). Which characteristics place the woman in a high risk category? a.Blood pressure, age, BMI b.Drug/alcohol use, age, family history c.Family history, blood pressure, BMI d.Family history, BMI, drug/alcohol abuse

ANS: D Her family history of NTD, low BMI, and substance abuse all are high risk factors of pregnancy. The womans BP is normal, and her age does not put her at risk. Her BMI is low and may indicate poor nutritional status, which would be a high risk. The womans drug/alcohol use and family history put her in a high risk category, but her age does not. The womans family history puts her in a high risk category. Her BMI is low and may indicate poor nutritional status, which would be high risk. Her BP is normal.

Which statement correctly describes the effects of various pain factors? a.Higher prostaglandin levels arising from dysmenorrhea can blunt the pain of childbirth. b.Upright positions in labor increase the pain factor because they cause greater fatigue. c.Women who move around trying different positions are experiencing more pain. d.Levels of pain-mitigating b-endorphins are higher during a spontaneous, natural childbirth.

ANS: D Higher endorphin levels help women tolerate pain and reduce anxiety and irritability. Higher prostaglandin levels correspond to more severe labor pains. Upright positions in labor usually result in improved comfort and less pain. Moving freely to find more comfortable positions is important for reducing pain and muscle tension.

Nurses should be aware that chronic hypertension: a.Is defined as hypertension that begins during pregnancy and lasts for the duration of pregnancy. b.Is considered severe when the systolic blood pressure (BP) is greater than 140 mm Hg or the diastolic BP is greater than 90 mm Hg. c.Is general hypertension plus proteinuria. d.Can occur independently of or simultaneously with gestational hypertension.

ANS: D Hypertension is present before pregnancy or diagnosed before 20 weeks of gestation and persists longer than 6 weeks postpartum. The range for hypertension is systolic BP greater than 140 mm Hg or diastolic BP greater than 90 mm Hg. It becomes severe with a diastolic BP of 110 mm Hg or higher. Proteinuria is an excessive concentration of protein in the urine. It is a complication of hypertension, not a defining characteristic.

A woman with preeclampsia has a seizure. The nurses primary duty during the seizure is to: a.Insert an oral airway. b.Suction the mouth to prevent aspiration. c.Administer oxygen by mask. d.Stay with the client and call for help.

ANS: D If a client becomes eclamptic, the nurse should stay her and call for help. Insertion of an oral airway during seizure activity is no longer the standard of care. The nurse should attempt to keep the airway patent by turning the clients head to the side to prevent aspiration. Once the seizure has ended, it may be necessary to suction the clients mouth. Oxygen would be administered after the convulsion has ended.

Nurses should be aware of the strengths and limitations of various biochemical assessments during pregnancy, including that: a.Chorionic villus sampling (CVS) is becoming more popular because it provides early diagnosis. b.Maternal serum alpha-fetoprotein (MSAFP) screening is recommended only for women at risk for neural tube defects c.Percutaneous umbilical blood sampling (PUBS) is one of the triple-marker tests for Down syndrome. d.MSAFP is a screening tool only; it identifies candidates for more definitive procedures.

ANS: D MSAFP is a screening tool, not a diagnostic tool. CVS provides a rapid result, but it is declining in popularity because of advances in noninvasive screening techniques. MSAFP screening is recommended for all pregnant women. MSAFP screening, not PUBS, is part of the triple-marker tests for Down syndrome.

In order to evaluate the condition of the patient accurately during labor, the nurse should be aware that: a.The womans blood pressure will increase during contractions and fall back to prelabor normal between contractions. b.Use of the Valsalva maneuver is encouraged during the second stage of labor to relieve fetal hypoxia. c.Having the woman point her toes will reduce leg cramps. d.The endogenous endorphins released during labor will raise the womans pain threshold and produce sedation.

ANS: D The endogenous endorphins released during labor will raise the womans pain threshold and produce sedation. In addition, physiologic anesthesia of the perineal tissues, caused by the pressure of the presenting part, decreases the mothers perception of pain. Blood pressure increases during contractions but remains somewhat elevated between them. Use of the Valsalva maneuver is discouraged during second stage labor because of a number of unhealthy outcomes, including fetal hypoxia. Pointing the toes can cause leg cramps, as can the process of labor itself.

The factors that affect the process of labor and birth, known commonly as the five Ps, include all except: a.Passenger. b.Passageway. c.Powers. d.Pressure.

ANS: D The five Ps are passenger (fetus and placenta), passageway (birth canal), powers (contractions), position of the mother, and psychologic response.

A new mother asks the nurse when the soft spot on her sons head will go away. The nurses answer is based on the knowledge that the anterior fontanel closes after birth by _____ months. A.2 b.8 c.12 d.18

ANS: D The larger of the two fontanels, the anterior fontanel, closes by 18 months after birth.

Nutrition is one of the most significant factors influencing the outcome of a pregnancy. It is an alterable and important preventive measure for various potential problems, such as low birth weight and prematurity. While completing the physical assessment of the pregnant client, the nurse can evaluate the clients nutritional status by observing a number of physical signs. Which sign would indicate that the client has unmet nutritional needs? a.Normal heart rate, rhythm, and blood pressure b.Bright, clear, shiny eyes c.Alert, responsive, and good endurance d.Edema, tender calves, and tingling

ANS: D The physiologic changes of pregnancy may complicate the interpretation of physical findings. Lower extremity edema often occurs when caloric and protein deficiencies are present; however, it may also be a common physical finding during the third trimester. It is essential that the nurse complete a thorough health history and physical assessment and request further laboratory testing if indicated. A malnourished pregnant patient may display rapid heart rate, abnormal rhythm, enlarged heart, and elevated blood pressure. A patient receiving adequate nutrition has bright, shiny eyes with no sores and moist, pink membranes. Pale or red membranes, dryness, infection, dull appearance of the cornea, or blue sclerae all are signs of poor nutrition. This client is well nourished. Cachexia, listlessness, and tiring easily would be indications of poor nutritional status.

A 22-year-old woman pregnant with a single fetus has a preconception body mass index (BMI) of 24. When she was seen in the clinic at 14 weeks of gestation, she had gained 1.8 kg (4 lb) since conception. How would the nurse interpret this? a.This weight gain indicates possible gestational hypertension b.This weight gain indicates that the womans infant is at risk for intrauterine growth restriction (IUGR). c.This weight gain cannot be evaluated until the woman has been observed for several more weeks. d.The womans weight gain is appropriate for this stage of pregnancy.

ANS: D The statement The womans weight gain is appropriate for this stage of pregnancy is accurate. This womans BMI is within the normal range. During the first trimester, the average total weight gain is only 1 to 2.5 kg. Although weight gain does indicate possible gestational hypertension, it does not apply to this patient. The desirable weight gain during pregnancy varies among women. The primary factor to consider in making a weight gain recommendation is the appropriateness of the prepregnancy weight for the womans height. A commonly used method of evaluating the appropriateness of weight for height is the BMI. Although weight gain does indicate risk for IUGR, this does not apply to this patient. Weight gain should occur at a steady rate throughout the pregnancy. The optimal rate of weight gain also depends on the stage of the pregnancy.

In comparing the abdominal and transvaginal methods of ultrasound examination, nurses should explain to their clients that: a.Both require the woman to have a full bladder. b.The abdominal examination is more useful in the first trimester. c.Initially the transvaginal examination can be painful. d.The transvaginal examination allows pelvic anatomy to be evaluated in greater detail.

ANS: D The transvaginal examination allows pelvic anatomy to be evaluated in greater detail and allows intrauterine pregnancies to be diagnosed earlier. The abdominal examination requires a full bladder; the transvaginal examination requires an empty bladder. The transvaginal examination is more useful in the first trimester; the abdominal examination works better after the first trimester. Neither method should be painful, although with the transvaginal examination the woman feels pressure as the probe is moved.

A primigravida at 39 weeks of gestation is observed for 2 hours in the intrapartum unit. The fetal heart rate has been normal. Contractions are 5 to 9 minutes apart, 20 to 30 seconds in duration, and of mild intensity. Cervical dilation is 1 to 2 cm and uneffaced (unchanged from admission). Membranes are intact. The nurse should expect the woman to be: a.Admitted and prepared for a cesarean birth. b.Admitted for extended observation. c.Discharged home with a sedative. d.Discharged home to await the onset of true labor.

ANS: D This situation describes a woman with normal assessments who is probably in false labor and will probably not deliver rapidly once true labor begins. These are all indications of false labor without fetal distress. There is no indication that further assessment or cesarean birth is indicated. The patient will likely be discharged; however, there is no indication that a sedative is needed.

Informed consent concerning contraceptive use is important because some of the methods: a.Are invasive procedures that require hospitalization b.Require a surgical procedure to insert c.May not be reliable d.Have potentially dangerous side effects

ANS: D To make an informed decision about the use of contraceptives, it is important for couples to be aware of potential side effects. The only contraceptive method that is a surgical procedure and requires hospitalization is sterilization. Some methods have greater efficacy than others, and this should be included in the teaching.

Maternity nurses often have to answer questions about the many, sometimes unusual ways people have tried to make the birthing experience more comfortable. For instance, nurses should be aware that: a.Music supplied by the support person has to be discouraged because it could disturb others or upset the hospital routine. b.Women in labor can benefit from sitting in a bathtub, but they must limit immersion to no longer than 15 minutes at a time. c.Effleurage is permissible, but counterpressure is almost always counterproductive. d.Electrodes attached to either side of the spine to provide high-intensity electrical impulses facilitate the release of endorphins

ANS: D Transcutaneous electrical nerve stimulation does help. Music may be very helpful for reducing tension and certainly can be accommodated by the hospital. Women can stay in a bath as long as they want, although repeated baths with breaks may be more effective than a long soak. Counterpressure can help the woman cope with lower back pain.

A 40-year-old woman is 10 weeks pregnant. Which diagnostic tool would be appropriate to suggest to her at this time? a.Biophysical profile (BPP) b.Amniocentesis c.Maternal serum alpha-fetoprotein (MSAFP) screening d.Transvaginal ultrasound

ANS: D Ultrasound would be performed at this gestational age for biophysical assessment of the infant. BPP would be a method of biophysical assessment of fetal well-being in the third trimester. Amniocentesis is performed after the fourteenth week of pregnancy. MSAFP screening is performed from week 15 to week 22 of gestation (weeks 16 to 18 are ideal).

A man smokes two packs of cigarettes a day. He wants to know if smoking is contributing to the difficulty he and his wife are having getting pregnant. The nurses most appropriate response is: a.Your sperm count seems to be okay in the first semen analysis. b.Only marijuana cigarettes affect sperm count. c.Smoking can give you lung cancer, even though it has no effect on sperm. d.Smoking can reduce the quality of your sperm.

ANS: D Use of tobacco, alcohol, and marijuana may affect sperm counts. Your sperm count seems to be okay in the first semen analysis is inaccurate. Sperm counts vary from day to day and depend on emotional and physical status and sexual activity. A single analysis may be inconclusive. A minimum of two analyses must be performed several weeks apart to assess male fertility.

A woman at 39 weeks of gestation with a history of preeclampsia is admitted to the labor and birth unit. She suddenly experiences increased contraction frequency of every 1 to 2 minutes; dark red vaginal bleeding; and a tense, painful abdomen. The nurse suspects the onset of: a.Eclamptic seizure. b.Rupture of the uterus. c.Placenta previa. d.Placental abruption.

ANS: D Uterine tenderness in the presence of increasing tone may be the earliest finding of premature separation of the placenta (abruptio placentae or placental abruption). Women with hypertension are at increased risk for an abruption. Eclamptic seizures are evidenced by the presence of generalized tonic-clonic convulsions. Uterine rupture manifests as hypotonic uterine activity, signs of hypovolemia, and in many cases the absence of pain. Placenta previa manifests with bright red, painless vaginal bleeding.


Ensembles d'études connexes

Identify the research method used and outline two characteristics of the method and Suggest an alternative or additional research method giving one reason for your choice.

View Set

Property and Casualty: Terms and Concepts

View Set

Chapter 24 - The Digestive System TB

View Set

Week 2: Check Your Understanding

View Set

Community Health Assessment A and B

View Set